Peds Exam 1: Ch 1, 2, 4, 19, 21, 23

¡Supera tus tareas y exámenes ahora con Quizwiz!

A nurse is teaching nursing students the physiology of congenital heart defects. Which defect results in decreased pulmonary blood flow? a. Atrial septal defect b. Tetralogy of Fallot c. Ventricular septal defect d. Patent ductus arteriosus

ANS: B Tetralogy of Fallot results in decreased blood flow to the lungs. The pulmonic stenosis increases the pressure in the right ventricle, causing the blood to go from right to left across the ventricular septal defect. Atrial and ventricular septal defects and patent ductus arteriosus result in increased pulmonary blood flow.

The nurse is taking a health history on an adolescent. Which best describes how the chief complaint should be determined? a. Ask for detailed listing of symptoms. b. Ask adolescent, "Why did you come here today?" c. Use what adolescent says to determine, in correct medical terminology, what the problem is. d. Interview parent away from adolescent to determine chief complaint.

ANS: B The chief complaint is the specific reason for the child's visit to the clinic, office, or hospital. Because the adolescent is the focus of the history, this is an appropriate way to determine the chief complaint. A detailed listing of symptoms will make it difficult to determine the chief complaint. The adolescent should be prompted to tell which symptom caused him to seek help at this time. The chief complaint is usually written in the words that the parent or adolescent uses to describe the reason for seeking help. The parent and adolescent may be interviewed separately, but the nurse should determine the reason the adolescent is seeking attention at this time.

Which factor most impacts the type of injury a child is susceptible to, according to the child's age? a. Physical health of the child b. Developmental level of the child c. Educational level of the child d. Number of responsible adults in the home

ANS: B The child's developmental stage determines the type of injury that is likely to occur. The child's physical health may facilitate the child's recovery from an injury but does not impact the type of injury. Educational level is related to developmental level, but it is not as important as the child's developmental level in determining the type of injury. The number of responsible adults in the home may affect the number of unintentional injuries, but the type of injury is related to the child's developmental stage.

The nurse encourages the mother of a toddler with acute laryngotracheobronchitis (LTB) to stay at the bedside as much as possible. The nurse's rationale for this action is described primarily in which statement? a. Mothers of hospitalized toddlers often experience guilt. b. The mother's presence will reduce anxiety and ease the child's respiratory efforts. c. Separation from the mother is a major developmental threat at this age. d. The mother can provide constant observations of the child's respiratory efforts.

ANS: B The family's presence will decrease the child's distress. It is true that mothers of hospitalized toddlers often experience guilt but this is not the best answer. The main reason to keep parents at the child's bedside is to ease anxiety and therefore respiratory effort. The child should have constant monitoring by cardiorespiratory monitor and noninvasive oxygen saturation monitoring, but the parent should not play this role in the hospital.

A 5-year-old girl is having a checkup before starting kindergarten. The nurse asks her to do the "finger-to-nose" test. What is the nurse testing for? a. Deep tendon reflexes b. Cerebellar function c. Sensory discrimination d. Ability to follow directions

ANS: B The finger-to-nose test is an indication of cerebellar function. This test checks balance and coordination. Each deep tendon reflex is tested separately. Each sense is tested separately. Although this test enables the nurse to evaluate the child's ability to follow directions, it is used primarily for cerebellar function.

The nurse is caring for an adolescent who had an external fixator placed after suffering a fracture of the wrist during a bicycle accident. Which statement by the adolescent should be expected about separation anxiety? a. "I wish my parents could spend the night with me while I am in the hospital." b. "I think I would like for my siblings to visit me but not my friends." c. "I hope my friends don't forget about visiting me." d. "I will be embarrassed if my friends come to the hospital to visit."

ANS: C Loss of peer-group contact may pose a severe emotional threat to an adolescent because of loss of group status, so friends visiting are an important aspect of hospitalization for an adolescent. Most adolescents do not need a parent to spend the night during hospitalization and sometimes view the hospitalization as a welcome event. Adolescents would be more concerned about friends visiting than siblings. Adolescents want visitors to keep control and maintain social status among their group of peers.

What is the primary nursing intervention to prevent bacterial endocarditis? a. Institute measures to prevent dental procedures. b. Counsel parents of high-risk children about prophylactic antibiotics. c. Observe children for complications, such as embolism and heart failure. d. Encourage restricted mobility in susceptible children.

ANS: B The objective of nursing care is to counsel the parents of high-risk children about both the need for prophylactic antibiotics for dental procedures and the necessity of maintaining excellent oral health. The child's dentist should be aware of the child's cardiac condition. Dental procedures should be done to maintain a high level of oral health. Prophylactic antibiotics are necessary. Children should be observed for complications such as embolism and heart failure and restricted mobility should be encouraged in susceptible children, but maintaining good oral health and prophylactic antibiotics is important.

Which is most important to document about immunizations in the child's health history? a. Dosage of immunizations received b. Occurrence of any reaction after an immunization c. The exact date the immunizations were received d. Practitioner who administered the immunizations

ANS: B The occurrence of any reaction after an immunization was given is the most important to document in a history because of possible future reactions, especially allergic reactions. Exact dosage of the immunization received may not be recorded on the immunization record. Exact dates are important to obtain but not as important as a history of reaction to an immunization. The practitioner who administered the immunization does not need to be recorded in the health history. A potentially severe physiologic response is the most threatening and most important information to document for safety reasons.

The nurse must check vital signs on a 2-year-old boy who is brought to the clinic for his 24-month checkup. What criteria should the nurse use in determining the appropriate-size blood pressure cuff? (Select all that apply.) a. The cuff is labeled "toddler." b. The cuff bladder width is approximately 40% of the circumference of the upper arm. c. The cuff bladder length covers 80% to 100% of the circumference of the upper arm. d. The cuff bladder covers 50% to 66% of the length of the upper arm.

ANS: B, C Research has demonstrated that cuff selection with a bladder width that is 40% of the arm circumference will usually have a bladder length that is 80% to 100% of the upper arm circumference. This size cuff will most accurately reflect measured radial artery pressure. The name of the cuff is a representative size that may not be suitable for any individual child. Choosing a cuff by limb circumference more accurately reflects arterial pressure than choosing a cuff by length.

A nurse plans therapeutic play time for a hospitalized child. What are the benefits of play? (Select all that apply.) a. Serves as method to assist disturbed children b. Allows the child to express feelings c. The nurse can gain insight into the child's feelings. d. The child can deal with concerns and feelings. e. Gives the child a structured play environment

ANS: B, C, D Play is an effective, nondirective modality for helping children deal with their concerns and fears, and at the same time, it often helps the nurse gain insights into children's needs and feelings. Play and other expressive activities provide one of the best opportunities for encouraging emotional expression, including the safe release of anger and hostility. Nondirective play that allows children freedom for expression can be tremendously therapeutic. Play therapy is a structured therapy that helps disturbed children. It should not be confused with therapeutic play.

A nurse is interviewing the parents of a toddler about use of complementary or alternative medical practices. The parents share several practices they use in their household. Which should the nurse document as complementary or alternative medical practices? (Select all that apply.) a. Use of acetaminophen (Tylenol) for fever b. Administration of chamomile tea at bedtime c. Hypnotherapy for relief of pain d. Acupressure to relieve headaches e. Cool mist vaporizer at the bedside for "stuffiness"

ANS: B, C, D When conducting an assessment, the nurse should inquire about the use of complementary or alternative medical practices. Administration of chamomile tea at bedtime, hypnotherapy for relief of pain, and acupressure to relieve headaches are complementary or alternative medical practices. Using Tylenol for fever relief and a cool mist vaporizer at the bedside to reduce "stuffiness" are not considered complementary or alternative medical practices.

A nurse is conducting a teaching session on the use of time-out as a discipline measure to parents of toddlers. Which are correct strategies the nurse should include in the teaching session? (Select all that apply.) a. Time-out as a discipline measure cannot be used when in a public place. b. A rule for the length of time-out is 1 minute per year. c. When the child misbehaves, one warning should be given. d. The area for time-out can be in the family room where the child can see the television. e. When the child is quiet for the specified time, he or she can leave the room.

ANS: B, C, E A rule for the length of time-out is 1 minute per year of age; use a kitchen timer with an audible bell to record the time rather than a watch. When the child misbehaves, one warning should be given. When the child is quiet for the duration of the time, he or she can then leave the room. Time-out can be used in public places and the parents should be consistent on the use of time-out. Implement time-out in a public place by selecting a suitable area or explain to children that time-out will be spent immediately on returning home. The time-out should not be spent in an area from which the child can view the television. Select an area for time-out that is safe, convenient, and unstimulating but where the child can be monitored, such as the bathroom, hallway, or laundry room.

Which behaviors by the nurse indicate a therapeutic relationship with children and families? (Select all that apply.) a. Spending off-duty time with children and families b. Asking questions if families are not participating in the care c. Clarifying information for families d. Buying toys for a hospitalized child e. Learning about the family's religious preferences

ANS: B, C, E Asking questions if families are not participating in the care, clarifying information for families, and learning about the family's religious preferences are positive actions and foster therapeutic relationships with children and families. Spending off-duty time with children and families and buying toys for a hospitalized child are negative actions and indicate overinvolvement with children and families, which is nontherapeutic.

Which behaviors by the nurse indicate therapeutic nurse-family boundaries? (Select all that apply.) a. Nurse visits family on days off. b. House rules are negotiated. c. Nurse buys child expensive gifts. d. Communication is open and two-way

ANS: B, D A home care nurse can establish therapeutic nurse-family boundaries by negotiating house rules and ensuring that communication is open and two-way. Visiting the family of off-duty days and buying expensive gifts for the child would be boundary crossing and nontherapeutic.

Dunst, Trivette, and Deal identified the qualities of strong families that help them function effectively. Which qualities are included? (Select all that apply.) a. Ability to stay connected without spending time together b. Clear set of family values, rules, and beliefs c. Adoption of one coping strategy that always promotes positive functioning in dealing with life events d. Sense of commitment toward growth of individual family members as opposed to that of the family unit e. Ability to engage in problem-solving activities f. Sense of balance between the use of internal and external family resources

ANS: B, E, F A clear set of family rules, values, and beliefs that establishes expectations about acceptable and desired behavior is one of the qualities of strong families that help them function effectively. Strong families also are able to engage in problem-solving activities and to find a balance between internal and external forces. Strong families have a sense of congruence among family members regarding the value and importance of assigning time and energy to meet needs. Strong families also use varied coping strategies. The sense of commitment is toward the growth and well-being of individual family members, as well as the family unit.

When is it generally recommended that a child with acute streptococcal pharyngitis may return to school? a. When sore throat is better. b. If no complications develop. c. After taking antibiotics for 24 hours. d. After taking antibiotics for 3 days.

ANS: C After children have taken antibiotics for 24 hours, they are no longer contagious to other children. Sore throat may persist longer than 24 hours after beginning antibiotic therapy, but the child is no longer considered contagious. Complications may take days to weeks to develop

A preschool child is scheduled for an echocardiogram. Parents ask the nurse whether they can hold the child during the procedure. The nurse should answer with which response? a. "You will be able to hold your child during the procedure." b. "Your child can be active during the procedure, but can't sit in your lap." c. "Your child must lie quietly; sometimes a mild sedative is administered before the procedure." d. "The procedure is invasive so your child will be restrained during the echocardiogram."

ANS: C Although an echocardiogram is noninvasive, painless, and associated with no known side effects, it can be stressful for children. The child must lie quietly in the standard echocardiographic positions; crying, nursing, or sitting up often leads to diagnostic errors or omissions. Therefore, infants and young children may need a mild sedative; older children benefit from psychological preparation for the test. The distraction of a video or movie is often helpful.

A nurse is assessing a patient admitted for an asthma exacerbation. Which breath sounds does the nurse expect to assess? a. Rubs b. Rattles c. Wheezes d. Crackles

ANS: C Asthma causes bronchoconstriction and narrowed passageways. Wheezes are produced as air passes through narrowed passageways. Rubs are the sound created by the friction of one surface rubbing over another. Pleural friction rub is caused by inflammation of the pleural space. Rattles is the term formerly used for crackles. Crackles are the sounds made when air passes through fluid or moisture.

Which defect results in increased pulmonary blood flow? a. Pulmonic stenosis b. Tricuspid atresia c. Atrial septal defect d. Transposition of the great arteries

ANS: C Atrial septal defect results in increased pulmonary blood flow. Blood flows from the left atrium (higher pressure) into the right atrium (lower pressure) and then to the lungs via the pulmonary artery. Pulmonic stenosis is an obstruction to blood flowing from the ventricles. Tricuspid atresia results in decreased pulmonary blood flow. Transposition of the great arteries results in mixed blood flow.

A nurse manager at a home-care agency is planning a continuing education program for the home-care staff nurses. Which type of continuing education program should the nurse manager plan? a. On-line training modules b. A structured written teaching module each nurse completes individually c. A workshop training day, with a professional speaker, where nurses can interact with each other d. One-on-one continuing education training with each nurse

ANS: C Because of the unique practice environment of home care nurses, it is important for an agency to facilitate sharing among peers to decrease work-related stress, increase job satisfaction, and support high-quality patient care. On-line training, written teaching modules, and one-on-one training would not allow for any sharing with peers.

While caring for a critically ill child, the nurse observes that respirations are gradually increasing in rate and depth, with periods of apnea. What pattern of respiration will the nurse document? a. Dyspnea b. Tachypnea c. Cheyne-Stokes respirations d. Seesaw (paradoxic) respirations

ANS: C Cheyne-Stokes respirations are a pattern of respirations that gradually increase in rate and depth, with periods of apnea. Dyspnea is defined as distress during breathing. Tachypnea is an increased respiratory rate. In seesaw respirations, the chest falls on inspiration and rises on expiration.

A nurse is counseling parents of a child beginning to show signs of being overweight. The nurse accurately relates which body mass index (BMI)-for-age percentile indicates a risk for being overweight? a. 10th percentile b. 9th percentile c. 85th percentile d. 95th percentile

ANS: C Children who have BMI-for-age greater than or equal to the 85th percentile and less than the 95th percentile are at risk for being overweight. Children in the 9th and 10th percentiles are within normal limits. Children who are greater than or equal to the 95th percentile are considered overweight.

Which is most likely to encourage parents to talk about their feelings related to their child's illness? a. Be sympathetic. b. Use direct questions. c. Use open-ended questions. d. Avoid periods of silence.

ANS: C Closed-ended questions should be avoided when attempting to elicit parents' feelings. Open-ended questions require the parent to respond with more than a brief answer. Sympathy is having feelings or emotions in common with another person rather than understanding those feelings (empathy). Sympathy is not therapeutic in helping the relationship. Direct questions may obtain limited information. In addition, the parent may consider them threatening. Silence can be an effective interviewing tool. It allows sharing of feelings in which two or more people absorb the emotion in depth. Silence permits the interviewee to sort out thoughts and feelings and search for responses to questions.

The home health nurse asks a child's mother many questions as part of the assessment. The mother answers many questions, then stops and says, "I don't know why you ask me all this. Who gets to know this information?" The nurse should take which action? a. Determine why the mother is so suspicious. b. Determine what the mother does not want to tell. c. Explain who will have access to the information. d. Explain that everything is confidential and that no one else will know what is said.

ANS: C Communication with the family should not be invasive. The nurse needs to explain the importance of collecting the information, its applicability to the child's care, and who will have access to the information. The mother is not being suspicious and is not necessarily withholding important information. She has a right to understand how the information she provides will be used. The nurse will need to share, through both oral and written communication, clinically relevant information with other involved health professionals.

What is an appropriate screening test for hearing that can be administered by the nurse to a 5-year-old child? a. The Rinne test b. The Weber test c. Conventional audiometry d. Eliciting the startle reflex

ANS: C Conventional audiometry is a behavioral test that measures auditory thresholds in response to speech and frequency-specific stimuli presented through earphones. The Rinne and Weber tests measure bone conduction of sound. Eliciting the startle reflex may be useful in infants.

Which leading cause of death topic should the nurse emphasize to a group of African-American boys ranging in age from 15 to 19 years? a. Suicide b. Cancer c. Firearm homicide d. Occupational injuries

ANS: C Firearm homicide is the second overall cause of death in this age group and the leading cause of death in African-American males. Suicide is the third-leading cause of death in this population. Cancer, although a major health problem, is the fourth-leading cause of death in this age group. Occupational injuries do not contribute to a significant death rate for this age group.

A nurse notes that a 10-month-old infant has a larger head circumference than chest. The nurse interprets this as a normal finding because the head and chest circumference become equal at which age? a. 1 month b. 6 to 9 months c. 1 to 2 years d. to 3 years

ANS: C Head circumference begins larger than chest circumference. Between ages 1 and 2 years, they become approximately equal. Head circumference is larger than chest circumference before age 1. Chest circumference is larger than head circumference at to 3 years.

Because injuries are the most common cause of death and disability in children in the United States, which stage of development correctly determines the type of injury that may occur? Select all that apply. 1. A newborn may roll over and fall off an elevated surface 2. The need to conform and gain acceptance from his peers may make a child accept a dare 3. Toddlers who can run and climb may be susceptible to burns, falls, and collisions with objects 4. A preschooler may ride her two-wheel bike in a reckless manner 5. A crawling infant may aspirate due to the tendency to place objects in his mouth.

2, 3, & 5: Adolescence may conform to more risky behaviors due to desire for acceptance, toddlers are more curious therefore may explore more vicariously, infants usually put things in their mouth which predispose them to choking, poisoning, and aspirating.

The nurse is caring for a child after heart surgery. What should the nurse do if evidence of cardiac tamponade is found? a. Increase analgesia b. Apply warming blankets c. Immediately report this to physician d. Encourage child to cough, turn, and breathe deeply

ANS: C If evidence is noted of cardiac tamponade, which is blood or fluid in the pericardial space constricting the heart, the physician is notified immediately of this life-threatening complication. Increasing analgesia may be done before the physician drains the fluid, but the physician must be notified. Warming blankets are not indicated at this time. Encouraging the child to cough, turn, and breathe deeply should be deferred till after the evaluation by the physician.

Which is an important nursing consideration when suctioning a young child who has had heart surgery? a. Perform suctioning at least every hour. b. Suction for no longer than 30 seconds at a time. c. Administer supplemental oxygen before and after suctioning. d. Expect symptoms of respiratory distress when suctioning.

ANS: C If suctioning is indicated, supplemental oxygen is administered with a manual resuscitation bag before and after the procedure to prevent hypoxia. Suctioning should be done only as indicated, not on a routine basis. The child should be suctioned for no more than 5 seconds at one time. Symptoms of respiratory distress are avoided by using appropriate technique.

A 6-month-old infant is receiving digoxin (Lanoxin). The nurse should notify the practitioner and withhold the medication if the apical pulse is less than _______ beats/min. a. 60 b. 70 c. 90 to 110 d. 110 to 120

ANS: C If the 1-minute apical pulse is below 90 to 110 beats/min, the digoxin should not be given to a 6-month-old. Sixty beats/min is the cut-off for holding the digoxin dose in an adult; 70 beats/min is the determining heart rate to hold a dose of digoxin for an older child; 110 to 120 beats/min is an acceptable heart rate to administer digoxin to a 6-month-old.

A family you are caring for on the Pediatric unit asks you about nutrition for their baby. What facts will you want to include in this nutritional information? Select all that apply. a. Breastfeeding provides macronutrients and immunologic properties b. Eating preferences and attitudes related to food are established by family influences and culture c. Most children establish lifelong eating habits by 18 months d. During adolescence, parental influence diminishes and adolescents make food choices related to peer acceptability and sociability e. Due to the stress of returning to work, most mothers use this as a time to stop breastfeeding.

A, B, & D: Children obtain lifelong eating habits by 3 years of age and although mothers find it hard to continue to breastfeed when returning to work, pumping regularly can promote breastfeeding.

The newest nurse on the Pediatric unit is concerned about maintaining a professional distance in her relationship with a patient and the patients family. Which comment indicates the she needs more mentoring regarding her patient-nurse relationship? a. "I realize that caring for the child means I can visit them on my off days if they ask me" b." When the mother asks if I will care for her daughter every day, I explain that the assignments change based on the needs of the unit" c. "When the mother asks me questions about my family, I answer politely, but I offer only pertinent information" d. "I engage in multidisciplinary rounds and listen to the family's concerns"

A: Typically to maintain a professional attitude, a nurse should not develop a personal relationship with clients and families.

Which is the highest acceptable mg/dl level of low density lipoprotein (LDL) cholesterol for a child from a family with heart disease? (Record your answer in a whole number.)

ANS: 110 The low-density lipoproteins (LDLs) contain low concentrations of triglycerides, high levels of cholesterol, and moderate levels of protein. LDL is the major carrier of cholesterol to the cells. Cells use cholesterol for synthesis of membranes and steroid production. Elevated circulating LDL is a strong risk factor in cardiovascular disease. For children from families with a history of heart disease, the LDL should be <110.

A nurse is admitting a toddler to the hospital. The toddler is with both parents and is currently sitting comfortably on a parent's lap. The parents state they will need to leave for a brief period. Which type of nursing diagnosis should the nurse formulate for this child? a. Risk for anxiety b. Anxiety c. Readiness for enhanced coping d. Ineffective coping

ANS: A A potential problem is categorized as a risk. The toddler has a risk to become anxious when the parents leave. Nursing interventions will be geared toward reducing the risk. The child is not showing current anxiety or ineffective coping. The child is not at a point for readiness for enhanced coping, especially because the parents will be leaving.

A child is admitted to the hospital with asthma. Which assessment findings support this diagnosis? a. Nonproductive cough, wheezing b. Fever, general malaise c. Productive cough, rales d. Stridor, substernal retractions

ANS: A Asthma presents with a nonproductive cough and wheezing. Pneumonia appears with an acute onset, fever, and general malaise. A productive cough and rales would be indicative of pneumonia. Stridor and substernal retractions are indicative of croup.

A nurse is preparing to administer an angiotensin-converting enzyme (ACE) inhibitor. Which drug should the nurse administer? a. Captopril (Capoten) b. Furosemide (Lasix) c. Spironolactone (Aldactone) d. Chlorothiazide (Diuril)

ANS: A Captopril is an ACE inhibitor. Furosemide is a loop diuretic. Spironolactone blocks the action of aldosterone. Chlorothiazide works on the distal tubules.

A nurse is interpreting the results of a tuberculin skin test (TST) on an adolescent who is HIV positive. Which induration size indicates a positive result for this child 48 to 72 hours after the test? a. ≥5 mm b. ≥10 mm c. ≥15 mm d. ≥20 mm

ANS: A Clinical evidence of a positive TST in children receiving immunosuppressive therapy, including immunosuppressive doses of steroids or who have immunosuppressive conditions, including HIV infection is an induration of ≥5 mm. Children younger than 4 years of age: (a) with other medical risk conditions, including Hodgkin disease, lymphoma, diabetes mellitus, chronic renal failure, or malnutrition; (b) born or whose parents were born in high-prevalence (TB) regions of the world; (c) frequently exposed to adults who are HIV infected, homeless, users of illicit drugs, residents of nursing homes, incarcerated or institutionalized, or migrant farm workers; and (d) who travel to high-prevalence (TB) regions of the world are positive when the induration is ≥10 mm. Children 4 years of age or older without any risk factors are positive when the induration is ≥20 mm.

Four-year-old Brian appears to be upset by hospitalization. Which is an appropriate intervention? a. Let him know it is all right to cry. b. Give him time to gain control of himself. c. Show him how other children are cooperating. d. Tell him what a big boy he is to be so quiet.

ANS: A Crying is an appropriate behavior for the upset preschooler. The nurse provides support through physical presence. Giving the child time to gain control is appropriate, but the child must know that crying is acceptable. The preschooler does not engage in competitive behaviors.

The nurse is preparing to administer a dose of digoxin (Lanoxin) to a child in heart failure (HF). Which is a beneficial effect of administering digoxin (Lanoxin)? a. It decreases edema. b. It decreases cardiac output. c. It increases heart size. d. It increases venous pressure.

ANS: A Digoxin has a rapid onset and is useful for increasing cardiac output, decreasing venous pressure, and, as a result, decreasing edema. Cardiac output is increased by digoxin. Heart size and venous pressure are decreased by digoxin.

Effective lone-rescuer CPR on a 5-year-old child should include a. two breaths to every 30 chest compressions. b. two breaths to every 15 chest compressions. c. reassessment of the child after 50 cycles of compression and ventilation. d. reassessment of the child every 10 minutes that CPR continues.

ANS: A Lone-rescuer CPR is two breaths to 30 compressions for all ages until signs of recovery occur. Reassessment of the child should take place after 20 cycles or 1 minute.

The nurse is using calipers to measure skinfold thickness over the triceps muscle in a school-age child. What is the purpose of doing this? a. To measure body fat b. To measure muscle mass c. To determine arm circumference d. To determine accuracy of weight measurement

ANS: A Measurement of skinfold thickness is an indicator of body fat. Arm circumference is an indirect measure of muscle mass. The accuracy of weight measurement should be verified with a properly balanced scale. Body fat is just one indicator of weight.

Which painful, tender, pea-sized nodules may appear on the pads of the fingers or toes in bacterial endocarditis? a. Osler nodes b. Janeway lesions c. Subcutaneous nodules d. Aschoff nodes

ANS: A Osler nodes are red, painful, intradermal nodes found on pads of the phalanges in bacterial endocarditis. Janeway lesions are painless hemorrhagic areas on palms and soles in bacterial endocarditis. Subcutaneous nodules are nontender swellings, located over bony prominences, commonly found in rheumatic fever. Aschoff nodules are small nodules composed of cells and leukocytes found in the interstitial tissues of the heart in rheumatic myocarditis.

The Heimlich maneuver is recommended for airway obstruction in children older than _____ year(s). a. 1 b. 4 c. 8 d. 12

ANS: A The Heimlich maneuver is recommended for airway obstruction in children older than 1 year. In children younger than 1 year, back blows and chest thrusts are administered. The Heimlich maneuver can be used in children older than 1 year.

The nurse is admitting a child with rheumatic fever. Which therapeutic management should the nurse expect to implement? a. Administering penicillin b. Avoiding salicylates (aspirin) c. Imposing strict bed rest for 4 to 6 weeks d. Administering corticosteroids if chorea develops

ANS: A The goal of medical management is the eradication of the hemolytic streptococci. Penicillin is the drug of choice. Salicylates can be used to control the inflammatory process, especially in the joints, and reduce the fever and discomfort. Bed rest is recommended for the acute febrile stage, but it does not need to be strict. The chorea is transient and will resolve without treatment.

The nurse is talking to a parent of an infant with heart failure about feeding the infant. Which statement about feeding the child is correct? a. "You may need to increase the caloric density of your infant's formula." b. "You should feed your baby every 2 hours." c. "You may need to increase the amount of formula your infant eats with each feeding." d. "You should place a nasal oxygen cannula on your infant during and after each feeding."

ANS: A The metabolic rate of infants with heart failure is greater because of poor cardiac function and increased heart and respiratory rates. Their caloric needs are greater than those of the average infants, yet their ability to take in the calories is diminished by their fatigue. Infants with heart failure should be fed every 3 hours; a 2-hour schedule does not allow for enough rest, and a 4-hour schedule is too long. Fluids must be carefully monitored because of the heart failure. Infants do not require supplemental oxygen with feedings.

Which figure depicts a nurse performing a test for the triceps reflex?

ANS: A To test the triceps reflex, the child is placed supine, with the forearm resting over the chest and the triceps tendon is struck with the reflex hammer. The other figures depict tests for biceps reflex (slightly above the antecubital space) patellar (knee), and Achilles (behind the foot).

Divorced parents of a preschool child are asking whether their child will display any feelings or behaviors related to the effect of the divorce. The nurse is correct when explaining that the parents should be prepared for which types of behaviors? (Select all that apply.) a. Displaying fears of abandonment b. Verbalizing that he or she "is the reason for the divorce" c. Displaying fear regarding the future d. Ability to disengage from the divorce proceedings e. Engaging in fantasy to understand the divorce

ANS: A, B, E A child 3 to 5 years of age (preschool) may display fears of abandonment, verbalize feelings that he or she is the reason for the divorce, and engage in fantasy to understand the divorce. He or she would not be displaying fear regarding the future until school age, and the ability to disengage from the divorce proceedings would be characteristic of an adolescent.

What is an appropriate nursing intervention to minimize separation anxiety in a hospitalized toddler? a. Provide for privacy b. Encourage parents to room in c. Explain procedures and routines d. Encourage contact with children the same age

ANS: B A toddler experiences separation anxiety secondary to being separated from the parents. To avoid this, the parents should be encouraged to room in as much as possible. Maintaining routines and ensuring privacy are helpful interventions, but they would not substitute for the parents. Encouraging contact with children the same age would not substitute for having the parents present.

A nurse makes the decision to apply a topical anesthetic to a child's skin before drawing blood. Which ethical principle is the nurse demonstrating? a. Autonomy b. Beneficence c. Justice d. Truthfulness

ANS: B Beneficence is the obligation to promote the patient's well-being. Applying a topical anesthetic before drawing blood promotes reducing the discomfort of the venipuncture. Autonomy is the patient's right to be self-governing. Justice is the concept of fairness. Truthfulness is the concept of honesty.

Parents of a newborn are concerned because the infant's eyes often "look crossed" when the infant is looking at an object. The nurse's response is that this is normal based on the knowledge that binocularity is normally present by what age? a. 1 month b. 3 to 4 months c. 6 to 8 months d. 12 months

ANS: B Binocularity is usually achieved by ages 3 to 4 months. 1 month is too young. If binocularity is not achieved by ages 6 to 12 months, the child must be observed for strabismus.

The nurse is teaching nursing students about shock that occurs in children. What is one of the most frequent causes of hypovolemic shock in children? a. Sepsis b. Blood loss c. Anaphylaxis d. Congenital heart disease

ANS: B Blood loss is the most frequent cause of hypovolemic shock in children. Sepsis causes septic shock, which is overwhelming sepsis and circulating bacterial toxins. Anaphylactic shock results from extreme allergy or hypersensitivity to a foreign substance. Congenital heart disease contributes to hypervolemia, not hypovolemia.

Which tool measures body fat most accurately? a. Stadiometer b. Calipers c. Cloth tape measure d. Paper or metal tape measure

ANS: B Calipers are used to measure skinfold thickness, which is an indicator of body fat content. Stadiometers are used to measure height. Cloth tape measures should not be used because they can stretch. Paper or metal tape measures can be used for recumbent lengths and other body measurements that must be made.

The nurse is teaching nursing students about normal physiologic changes in the respiratory system of toddlers. Which best describes why toddlers have fewer respiratory tract infections as they grow older? a. The amount of lymphoid tissue decreases. b. Repeated exposure to organisms causes increased immunity. c. Viral organisms are less prevalent in the population. d. Secondary infections rarely occur after viral illnesses.

ANS: B Children have increased immunity after exposure to a virus. The amount of lymphoid tissue increases as children grow older. Viral organisms are not less prevalent, but older children have the ability to resist invading organisms. Secondary infections after viral illnesses include Mycoplasma pneumoniae and group A β-hemolytic streptococcal infections.

Which type of croup is always considered a medical emergency? a. Laryngitis b. Epiglottitis c. Spasmodic croup d. Laryngotracheobronchitis (LTB)

ANS: B Epiglottitis is always a medical emergency needing antibiotics and airway support for treatment. Laryngitis is a common viral illness in older children and adolescents, with hoarseness and URI symptoms. Spasmodic croup is treated with humidity. LTB may progress to a medical emergency in some children.

Parents of a firstborn child are asking whether it is normal for their child to be extremely competitive. The nurse should respond to the parents that studies about the ordinal position of children suggest that firstborn children tend to: a. be praised less often. b. be more achievement oriented. c. be more popular with the peer group. d. identify with peer group more than parents.

ANS: B Firstborn children, like only children, tend to be more achievement oriented. Being praised less often, being more popular with the peer group, and identifying with peer groups more than parents are characteristics of later-born children.

Which occurs in septic shock? a. Hypothermia b. Increased cardiac output c. Vasoconstriction d. Angioneurotic edema

ANS: B Increased cardiac output, which results in warm, flushed skin, is one of the manifestations of septic shock. Fever and chills are characteristic of septic shock. Vasodilation is more common than vasoconstriction. Angioneurotic edema occurs as a manifestation in anaphylactic shock.

The nurse is caring for a child with acute respiratory distress syndrome (ARDS) associated with sepsis. What is the priority nursing intervention? a. Forcing fluids b. Monitoring pulse oximetry c. Instituting seizure precautions d. Encouraging a high-protein diet

ANS: B Monitoring cardiopulmonary status is an important evaluation tool in the care of the child with ARDS. Maintenance of vascular volume and hydration is important and should be done parenterally. Seizures are not a side effect of ARDS. Adequate nutrition is necessary, but a high-protein diet is not helpful.

A child with cystic fibrosis (CF) is receiving recombinant human deoxyribonuclease (DNase). Which is an adverse effect of this medication? a. Mucus thickens b. Voice alters c. Tachycardia d. Jitteriness

ANS: B One of the only adverse effects of DNase is voice alterations and laryngitis. DNase decreases viscosity of mucus, is given in an aerosolized form, and is safe for children younger than 12 years. β2 Agonists can cause tachycardia and jitteriness.

A parent of a school-age child is going through a divorce. The parent tells the school nurse the child has not been doing well in school and sometimes has trouble sleeping. The nurse should recognize this as which implication? a. Indication of maladjustment b. Common reaction to divorce c. Lack of adequate parenting d. Unusual response that indicates need for referral

ANS: B Parental divorce affects school-age children in many ways. In addition to difficulties in school, they often have profound sadness, depression, fear, insecurity, frequent crying, loss of appetite, and sleep disorders. This is not an indication of maladjustment, suggestive of lack of adequate parenting, or an unusual response that indicates need for referral in school-age children after parental divorce.

What is the earliest age at which a satisfactory radial pulse can be taken in children? a. 1 year b. 2 years c. 3 years d. 6 years

ANS: B Satisfactory radial pulses can be used in children older than 2 years. In infants and young children, the apical pulse is more reliable. The apical pulse can be used for assessment at these ages.

A school nurse is conducting vision and hearing testing on fifth-grade children. Which level of prevention is the nurse demonstrating? a. Primary b. Secondary c. Tertiary d. Health promotion

ANS: B Secondary prevention focuses on screening and early diagnosis of disease. Vision and hearing testing are screening tests to detect problems. Primary prevention focuses on health promotion and prevention of disease or injury. Tertiary prevention focuses on optimizing function for children with a disability or chronic disease. Health promotion is focused on preventing disease or illness.

Where is the appropriate placement of a tongue blade for assessment of the mouth and throat? a. Center back area of tongue b. Side of the tongue c. Against the soft palate d. On the lower jaw

ANS: B Side of the tongue is the correct position. It avoids the gag reflex yet allows visualization. Placement in the center back area of the tongue will elicit the gag reflex. Against the soft palate and on the lower jaw are not appropriate places for the tongue blade.

Which is best described as the inability of the heart to pump an adequate amount of blood to the systemic circulation at normal filling pressures? a. Pulmonary congestion b. Congenital heart defect c. Heart failure d. Systemic venous congestion

ANS: C The definition of heart failure is the inability of the heart to pump an adequate amount of blood to the systemic circulation at normal filling pressures to meet the body's metabolic demands. Pulmonary congestion is an excessive accumulation of fluid in the lungs. Congenital heart defect is a malformation of the heart present at birth. Systemic venous congestion is an excessive accumulation of fluid in the systemic vasculature.

Where in the health history should the nurse describe all details related to the chief complaint? a. Past history b. Chief complaint c. Present illness d. Review of systems

ANS: C The history of the present illness is a narrative of the chief complaint from its earliest onset through its progression to the present. The focus of the present illness is on all factors relevant to the main problem, even if they have disappeared or changed during the onset, interval, and present. Past history refers to information that relates to previous aspects of the child's health, not to the current problem. The chief complaint is the specific reason for the child's visit to the clinic, office, or hospital. It does not contain the narrative portion describing the onset and progression. The review of systems is a specific review of each body system.

Which is the term for a family in which the paternal grandmother, the parents, and two minor children live together? a. Blended b. Nuclear c. Binuclear d. Extended

ANS: D An extended family contains at least one parent, one or more children, and one or more members (related or unrelated) other than a parent or sibling. A blended family contains at least one stepparent, step-sibling, or half-sibling. The nuclear family consists of two parents and their children. No other relatives or nonrelatives are present in the household. In binuclear families, parents continue the parenting role while terminating the spousal unit. For example, when joint custody is assigned by the court, each parent has equal rights and responsibilities for the minor child or children.

Which clinical manifestation should the nurse expect to see as shock progresses in a child and becomes decompensated shock? a. Thirst b. Irritability c. Apprehension d. Confusion and somnolence

ANS: D Confusion and somnolence are beginning signs of decompensated shock. Thirst, irritability, and apprehension are signs of compensated shock.

A nurse is selecting a family theory to assess a patient's family dynamics. Which family theory best describes a series of tasks for the family throughout its life span? a. Interactional theory b. Developmental systems theory c. Structural-functional theory d. Duvall's developmental theory

ANS: D Duvall's developmental theory describes eight developmental tasks of the family throughout its life span. Interactional theory and structural-functional theory are not family theories. Developmental systems theory is an outgrowth of Duvall's theory. The family is described as a small group, a semiclosed system of personalities that interact with the larger cultural system. Changes do not occur in one part of the family without changes in others.

When discussing hyperlipidemia with a group of adolescents, which high level labs should the nurse explain can prevent cardiovascular disease? a. Cholesterol b. Triglycerides c. Low-density lipoproteins (LDLs) d. High-density lipoproteins (HDLs)

ANS: D HDLs contain very low concentrations of triglycerides, relatively little cholesterol, and high levels of proteins. It is thought that HDLs protect against cardiovascular disease. Cholesterol, triglycerides, and LDLs are not protective against cardiovascular disease.

When caring for the child with Kawasaki disease, the nurse should know which information? a. A child's fever is usually responsive to antibiotics within 48 hours. b. The principal area of involvement is the joints. c. Aspirin is contraindicated. d. Therapeutic management includes administration of gamma globulin and aspirin.

ANS: D High-dose IV gamma globulin and aspirin therapy is indicated to reduce the incidence of coronary artery abnormalities when given within the first 10 days of the illness. The fever of Kawasaki disease is unresponsive to antibiotics and antipyretics. Mucous membranes, conjunctiva, changes in the extremities, and cardiac involvement are seen. Aspirin is part of the therapy.

The parent of an infant with nasopharyngitis should be instructed to notify the health professional if the infant displays which clinical manifestation? a. Fussiness b. Coughing c. A fever over 99° F d. Signs of an earache

ANS: D If an infant with nasopharyngitis shows signs of an earache, it may mean a secondary bacterial infection is present and the infant should be referred to a practitioner for evaluation. Irritability is common in an infant with a viral illness. Cough can be a sign of nasopharyngitis. Fever is common in viral illnesses.

During examination of a toddler's extremities, the nurse notes that the child is bowlegged. What should the nurse recognize regarding this finding? a. Abnormal and requires further investigation b. Abnormal unless it occurs in conjunction with knock-knee c. Normal if the condition is unilateral or asymmetric d. Normal because the lower back and leg muscles are not yet well developed

ANS: D Lateral bowing of the tibia (bowlegged) is common in toddlers when they begin to walk. It usually persists until all their lower back and leg muscles are well developed. Further evaluation is needed if it persists beyond ages 2 to 3 years, especially in African-American children.

José is a 4-year-old child scheduled for a cardiac catheterization. What should be included in preoperative teaching? a. Directed at his parents because he is too young to understand b. Detailed in regard to the actual procedures so he will know what to expect c. Done several days before the procedure so that he will be prepared d. Adapted to his level of development so that he can understand

ANS: D Preoperative teaching should always be directed at the child's stage of development. The caregivers also benefit from the same explanations. The parents may ask additional questions, which should be answered, but the child needs to receive the information based on developmental level. Preschoolers will not understand in-depth descriptions and should be prepared close to the time of the cardiac catheterization.

The nurse is using the Centers for Disease Control and Prevention (CDC) growth chart for an African-American child. Which statement should the nurse consider? a. This growth chart should not be used. b. Growth patterns of African-American children are the same as for all other ethnic groups. c. A correction factor is necessary when the CDC growth chart is used for non-Caucasian ethnic groups. d. The CDC charts are accurate for US African-American children.

ANS: D The CDC growth charts can serve as reference guides for all racial or ethnic groups. US African-American children were included in the sample population. The growth chart can be used with the perspective that different groups of children have varying normal distributions on the growth curves. No correction factor exists.

What does the surgical closure of the ductus arteriosus do? a. Stop the loss of unoxygenated blood to the systemic circulation b. Decrease the edema in legs and feet c. Increase the oxygenation of blood d. Prevent the return of oxygenated blood to the lungs

ANS: D The ductus arteriosus allows blood to flow from the higher-pressure aorta to the lower-pressure pulmonary artery, causing a right-to-left shunt. If this is surgically closed, no additional oxygenated blood (from the aorta) will return to the lungs through the pulmonary artery. The aorta carries oxygenated blood to the systemic circulation. Because of the higher pressure in the aorta, blood is shunted into the pulmonary artery and the pulmonary circulation. Edema in the legs and feet is usually a sign of heart failure. This repair would not directly affect the edema. Increasing the oxygenation of blood would not interfere with the return of oxygenated blood to the lungs.

Which is most characteristic of the physical punishment of children, such as spanking? a. Psychological impact is usually minimal. b. Children rarely become accustomed to spanking. c. Children's development of reasoning increases. d. Misbehavior is likely to occur when parents are not present.

ANS: D Through the use of physical punishment, children learn what they should not do. When parents are not around, it is more likely that children will misbehave because they have not learned to behave well for their own sake, but rather out of fear of punishment. Spanking can cause severe physical and psychological injury and interfere with effective parent-child interaction. Children do become accustomed to spanking, requiring more severe corporal punishment each time. The use of corporal punishment may interfere with the child's development of moral reasoning.

Which following parameters correlates best with measurements of the body's total protein stores? a. Height b. Weight c. Skinfold thickness d. Upper arm circumference

ANS: D Upper arm circumference is correlated with measurements of total muscle mass. Muscle serves as the body's major protein reserve and is considered an index of the body's protein stores. Height is reflective of past nutritional status. Weight is indicative of current nutritional status. Skinfold thickness is a measurement of the body's fat content.

When a preschool child is hospitalized without adequate preparation, how does the child often react to the hospitalization? a. Sees it as a punishment b. A threat to child's self-image c. An opportunity for regression d. Loss of companionship with friends

ANS: D When a child is hospitalized, the altered family role, physical disability, loss of peer acceptance, lack of productivity, and inability to cope with stress usurp individual power and identity. This is especially detrimental to school-age children, who are striving for independence and productivity and are now experiencing events that lessen their control and power. Infants, toddlers, and preschoolers, although affected to different extents by loss of power, are not as significantly affected as are school-age children.

Culture includes which of the following. Select all that apply. a. Cultural competence: which includes building skills in the healthcare provider, such as offering lists of common foods, health care beliefs, and important rituals. b. Cultural humility: which requires that healthcare providers participate in a continual process of self-reflection and self-critique c. Recognizing the power of the healthcare provider role that views the patient and family as full members of the healthcare team d. A particular group with its values, beliefs, norms, patterns, and practices that are learned, shared, and transmitted from one generation to another. e. A complex whole in which each part is interrelated, including beliefs, tradition, life ways, and heritage.

B, C, D, E: Cultural competence is not a part of culture although is important when working with individuals of another culture.

The overriding goal of atraumatic care is: a. Prevent or minimize the child's separation from the family b. Do no harm c. Promote a sense of control d. Prevent or minimize bodily injury and pain

B: Atraumatic care is the provision of therapeutic care through the use of interventions that eliminate or minimize the psychologic and physical distress experienced by children and families in the health care system

Family systems theory includes: a. Direct causality, meaning each change affects the whole family b. Family systems react to changes as they take place, not initiate it. c. A balance between morphogenesis and morphostasis is necessary d. Theory is used primarily for family dysfunction and pathology

C. Characterized by components of the systems and the environment. A change in one part of the family impacts the entire family.

The nurse is explaining the strategy of consequences to a parent he is working with. Which response by the parent indicated more teaching is needed when he describes the types of consequences? a. Natural: Those that occur without any intervention b. Logical: Those that are directly related to the rule c. Transforming: Allowing the child to come to the conclusion on his or her own d. Unrelated: Those are imposed deliberately

C: Transforming is not a type of consequence. Natural or logical consequences are the most preferred and effective.

Ways to integrate spiritual practices into nursing care include: a. Explaining the religious practices you personally take part in b. Realizing that young children have little understanding regarding their spirituality c. Agreeing with children and their families when they explain their religious beliefs so they are not offended d. Becoming knowledgable about the religious worldview of cultural groups found in the patients you care for.

D: Becoming knowledgable on cultural practices allows the nurse to provide the most competent and individualized care.

You are working with a new graduate on the pediatric unit and your patient is returning from the cardiac catheterization lab. You feel the graduate understands the important nursing interventions when she says which of the following? Select all that apply. a. "Check pulses especially below the catheterization site, for equality and symmetry." b. "Check vital signs, which may be taken as frequently as every 30 to 45 minutes, with special emphasis on the heart rate, which is counted for 1 full minute for evidence of dysrhythmias or bradycardia." c. "Special attention needs to be given to the BP, especially for hypertension, which may indicate hemorrhage or bleeding from the catheterization site." d. "Check the dressing for evidence of bleeding or hematoma formation in the femoral or antecubital area." e. "Allow the child to ambulate because this will prevent skin breakdown from lying so long in one place."

a. "Check pulses especially below the catheterization site, for equality and symmetry." d. "Check the dressing for evidence of bleeding or hematoma formation in the femoral or antecubital area."

You are working with a family with a child who has a congenital heart defect. Future surgery is planned, and you are teaching the parent how to reduce cardiac demands. The parent needs more teaching when she says which of the following? a. "I will wake my child for feeding every 2 hours so he can get enough calories to gain weight." b. "When I give the digoxin, I will listen to the pulse for 1 full minute." c. "I should protect my child from people who have respiratory infections." d. "I will count the number of wet diapers to be sure my child is not getting too much or too little fluid."

a. "I will wake my child for feeding every 2 hours so he can get enough calories to gain weight."

A 5-year-old is recovering from a tonsillectomy and adenoidectomy and is being discharged home with his mother. Home care instructions should include which of the following? Select all that apply. a. Observe the child for continuous swallowing. b. Encourage the child to take sips of cool, clear liquids. c. Administer codeine elixir as necessary for throat pain. d. Observe the child for restlessness or difficulty breathing. e. Encourage the child to cough every 4 to 5 hours to prevent pneumonia. f. Administer an analgesic such as acetaminophen for pain.

a. Observe the child for continuous swallowing. b. Encourage the child to take sips of cool, clear liquids. d. Observe the child for restlessness or difficulty breathing. f. Administer an analgesic such as acetaminophen for pain.

You are discharging a 5-week-old infant with a congenital heart defect who will be going home on digoxin. Which of the following answers by the father indicate the need for more teaching? Select all that apply. a. "I know I give the drug carefully by slowly directing it to the side and back of the mouth." b. "I give the medication every 12 hours, and I can place it in a bit of formula so that I know the baby will take it." c. "If I miss a dose, I don't give an extra dose, but I give the next dose as ordered." d. "If the baby vomits, I should give a second dose." e. "If more than two doses have been missed, I should call the doctor."

b. "I give the medication every 12 hours, and I can place it in a bit of formula so that I know the baby will take it." d. "If the baby vomits, I should give a second dose."

While orienting a new nurse to the ICU, she asks, "How do these children sleep and not become frightened with all the lights and noises?" How should you respond? Select all that apply. a. "These children are sicker than those on the pediatric unit, so the noises and lights are necessary." b. "We try to organize care into clusters so that infants and children can sleep and we can turn down lights." c. "We silence alarms to allow for periods of sleep, especially at night." d. "When possible, we allow for uninterrupted sleep cycles—for infants 90 minutes and for older children 60 minutes." e. "We encourage parents to sit with and touch their child as often as possible."

b. "We try to organize care into clusters so that infants and children can sleep and we can turn down lights." e. "We encourage parents to sit with and touch their child as often as possible."

You are working in the pediatric clinic, and a child presents with symptoms that are suspicious of the acute phase of Kawasaki disease. Which of the following symptoms are included? Select all that apply. a. Periungual desquamation (peeling that begins under the fingertips and toes) of the hands and feet is present. b. The bulbar conjunctivae of the eyes become reddened, with clearing around the iris. c. A temporary arthritis is evident, which may affect the larger weight-bearing joints. d. Inflammation of the pharynx and the oral mucosa develops, with red, cracked lips and the characteristic "strawberry tongue." e. Loud pansystolic murmur along with ECG changes are present.

b. The bulbar conjunctivae of the eyes become reddened, with clearing around the iris. d. Inflammation of the pharynx and the oral mucosa develops, with red, cracked lips and the characteristic "strawberry tongue."

A 5-year-old is seen in the urgent care clinic with the following history and symptoms: sudden onset of severe sore throat after going to bed, drooling and difficulty swallowing, axillary temperature of 102.2° F (39.0° C), clear breath sounds, and absence of cough. The child appears anxious and is flushed. Based on these symptoms and history, the nurse anticipates a diagnosis of: a. Group A beta-hemolytic streptococcus (GABHS) pharyngitis b. Acute tracheitis c. Acute epiglottitis d. Acute laryngotracheobronchitis (LTB)

c. Acute epiglottitis

A 12-year-old child is in the urgent care clinic with a complaint of fever, headache, and sore throat. A diagnosis of group A beta-hemolytic streptococcus (GABHS) pharyngitis is established with a rapid-strep test, and oral penicillin is prescribed. The nurse knows which of the following statements about GABHS is correct? a. Children with a GABHS infection are less likely to contract the illness again after the antibiotic regimen is completed. b. A follow-up throat culture is recommended following the completion of antibiotic therapy. c. Children with a GABHS infection are at increased risk for the development of rheumatic fever (RF) and glomerulonephritis. d. Children with a GABHS infection are at increased risk for the development of rheumatoid arthritis in adulthood.

c. Children with a GABHS infection are at increased risk for the development of rheumatic fever (RF) and glomerulonephritis.

A nurse looks over her assignment for the day that includes an infant, a preschool-age child, a third-grader, and a sophomore in high school. Which techniques take into consideration developmental stages when working with pediatric patients? a. Being aware that infants will become agitated due to stranger anxiety around 4 months old b. When a preschooler is getting blood drawn, giving a detailed explanation will be helpful c. Explaining and demonstrating what the BP machine does to the third-grader before taking her blood pressure d. Using a single consistent approach with the adolescent will help allay anger and hostility

c. Explaining and demonstrating what the BP machine does to the third-grader before taking her blood pressure

Separation anxiety is something that affects children when they are hospitalized. Each developmental stage has a somewhat different reaction as they deal with this difficulty. Which stage corresponds to the adolescent stage? a. May demonstrate separation anxiety by refusing to eat, experiencing difficulty in sleeping, crying quietly for their parents, continually asking when the parents will visit, or withdrawing from others. b. Separation anxiety comes in stages: protest, despair, and detachment. c. Loss of peer group contact may pose a severe emotional threat because of loss of group status, inability to exert group control or leadership, and loss of group acceptance. d. May need and desire parental guidance or support from other adult figures but may be unable or unwilling to ask for it.

c. Loss of peer group contact may pose a severe emotional threat because of loss of group status, inability to exert group control or leadership, and loss of group acceptance.

Which is the leading cause of death from unintentional injuries for females ranging in age from 1 to 14? a. Mechanical suffocation b. Drowning c. Motor vehicle-related fatalities d. Fire- and burn-related fatalities

ANS: C Motor vehicle-related fatalities are the leading cause of death for females ranging in age from 1 to 14, either as passengers or as pedestrians. Mechanical suffocation is fourth or fifth, depending on the age. Drowning is the second- or third-leading cause of death, depending on the age. Fire- and burn-related fatalities are the second-leading cause of death.

A nurse is preparing to perform a physical assessment on a toddler. Which approach should the nurse use for this child? a. Always proceed in a head-to-toe direction. b. Perform traumatic procedures first. c. Use minimal physical contact initially. d. Demonstrate use of equipment.

ANS: C Parents can remove clothing, and the child can remain on the parent's lap. The nurse should use minimal physical contact initially to gain the child's cooperation. The head-to-toe assessment can be done in older children but usually must be adapted in younger children. Traumatic procedures should always be performed last. These will most likely upset the child and inhibit cooperation. The nurse should introduce the equipment slowly. The child can inspect the equipment, but demonstrations are usually too complex for toddlers.

A nurse is caring for a child in acute respiratory failure. Which blood gas analysis indicates the child is still in respiratory acidosis? a. pH 7.50, CO2 48 b. pH 7.30, CO2 30 c. pH 7.32, CO2 50 d. pH 7.48, CO2 33

ANS: C Respiratory failure is a process that involves pulmonary dysfunction generally resulting in impaired alveolar gas exchange, which can lead to hypoxemia or hypercapnia. Acidosis indicates the pH is less than 7.35 and the CO2 is greater than 45. If the pH is less than 7.35 but the CO2 is low, it is metabolic acidosis. Alkalosis is when the pH is greater than 7.45. If the pH is high and the CO2 is high, it is metabolic alkalosis. When the pH is high and the CO2 is low, it is respiratory alkalosis.

A nurse is assigned to four children of different ages. In which age group should the nurse understand that body integrity is a concern? a. Toddler b. Preschooler c. School-age child d. Adolescent

ANS: C School-age children have a heightened concern about body integrity. They place importance and value on their bodies and are oversensitive to anything that constitutes a threat or suggestion of injury. Body integrity is not as important a concern to toddlers, preschoolers, or adolescents.

Which is a clinical manifestation of the systemic venous congestion that can occur with heart failure? a. Tachypnea b. Tachycardia c. Peripheral edema d. Pale, cool extremities

ANS: C Peripheral edema, especially periorbital edema, is a clinical manifestation of systemic venous congestion. Tachypnea is a manifestation of pulmonary congestion. Tachycardia and pale, cool extremities are clinical manifestations of impaired myocardial function.

Where is the best place to observe for the presence of petechiae in dark-skinned individuals? a. Face b. Buttocks c. Oral mucosa d. Palms and soles

ANS: C Petechiae, small distinct pinpoint hemorrhages, are difficult to see in dark skin unless they are in the mouth or conjunctiva.

A nurse is admitting a child, in foster care, to the hospital. The nurse recognizes that foster parents care for the child _____ hours a day. (Record your answer as a whole number.)

ANS: 24 The term foster care is defined as 24-hour substitute care for children outside of their own homes.

A nurse is interpreting the results of a child's peak expiratory flow rate. Which percentage, either at this number or less than this number, is considered to be a red zone? (Record your answer in a whole number.)

ANS: 50 A peak expiratory flow rate of red (<50% of personal best) signals a medical alert. Severe airway narrowing may be occurring. A short-acting bronchodilator should be administered. Notify the practitioner if the peak expiratory flow rate does not return immediately and stay in yellow or green zones.

The nurse enters a room and finds a 6-year-old child who is unconscious. After calling for help and before being able to use an automatic external defibrillator, which steps should the nurse take? Place in correct order. Provide answer using lowercase letters separated by commas (e.g., a, b, c, d, e, f). a. Place on a hard surface. b. Administer 30 chest compressions with two breaths. c. Feel carotid pulse while maintaining head tilt with the other hand. d. Use the head tilt-chin lift maneuver and check for breathing. e. Place heel of one hand on lower half of sternum with other hand on top. f. Give two rescue breaths.

ANS: a, d, f, c, e, b

An infant with an unrepaired tetralogy of Fallot defect is becoming extremely cyanotic during a routine blood draw. Which interventions should the nurse implement? Place in order from the highest-priority intervention to the lowest-priority intervention. Provide the answer using lowercase letters separated by commas (e.g., a, b, c, d). a. Administer 100% oxygen by blow-by. b. Place the infant in knee-chest position. c. Remain calm. d. Give morphine subcutaneously or by an existing intravenous line.

ANS: b, a, d, c

What is the correct sequence used when performing an abdominal assessment? Begin with the first technique and end with the last. Provide answer using lowercase letters separated by commas (e.g., a, b, c, d). a. Auscultation b. Palpation c. Inspection d. Percussion

ANS: c, a, d, b The correct order of abdominal examination is inspection, auscultation, percussion, and palpation. Palpation is always performed last because it may distort the normal abdominal sounds.

A nurse is formulating a clinical question for evidence-based practice. Place in order the steps the nurse should use to clarify the scope of the problem and clinical topic of interest. Begin with the first step of the process and proceed ordering the steps ending with the final step of the process. Provide answer as lowercase letters separated by commas (e.g., a, b, c, d, e). a. Intervention b. Outcome c. Population d. Time e. Control

ANS: c, a, e, b, d When formulating a clinical question for evidence-based practice, the nurse should follow a concise, organized way that allows for clear answers. Good clinical questions should be asked in the PICOT (population, intervention, control, outcome, time) format to assist with clarity and literature searching. PICOT questions assist with clarifying the scope of the problem and clinical topic of interest.

A nurse is assessing a family's structure. Which describes a family in which a mother, her children, and a stepfather live together? a. Blended b. Nuclear c. Binuclear d. Extended

ANS: A A blended family contains at least one stepparent, step-sibling, or half-sibling. The nuclear family consists of two parents and their children. No other relatives or nonrelatives are present in the household. In binuclear families, parents continue the parenting role while terminating the spousal unit. For example, when joint custody is assigned by the court, each parent has equal rights and responsibilities for the minor child or children. An extended family contains at least one parent, one or more children, and one or more members (related or unrelated) other than a parent or sibling.

During a funduscopic examination of a school-age child, the nurse notes a brilliant, uniform red reflex in both eyes. How should the nurse interpret this finding? a. Normal finding b. Abnormal finding, so child needs referral to ophthalmologist c. Sign of possible visual defect, so child needs vision screening d. Sign of small hemorrhages, which will usually resolve spontaneously

ANS: A A brilliant, uniform red reflex is an important normal finding. It rules out many serious defects of the cornea, aqueous chamber, lens, and vitreous chamber.

A child is being admitted to the hospital to be tested for cystic fibrosis (CF). Which tests should the nurse expect? a. Sweat chloride test, stool for fat, chest radiograph films b. Stool test for fat, gastric contents for hydrochloride, chest radiograph films c. Sweat chloride test, bronchoscopy, duodenal fluid analysis d. Sweat chloride test, stool for trypsin, biopsy of intestinal mucosa

ANS: A A sweat test result of greater than 60 mEq/L is diagnostic of CF, a high level of fecal fat is a gastrointestinal (GI) manifestation of CF, and a chest radiograph showing patchy atelectasis and obstructive emphysema indicates CF. Gastric contents contain hydrochloride normally; it is not diagnostic. Bronchoscopy and duodenal fluid are not diagnostic. Stool test for trypsin and intestinal biopsy are not helpful in diagnosing CF.

Which is most suggestive that a nurse has a nontherapeutic relationship with a patient and family? a. Staff is concerned about the nurse's actions with the patient and family. b. Staff assignments allow the nurse to care for same patient and family over an extended time. c. Nurse is able to withdraw emotionally when emotional overload occurs but still remains committed. d. Nurse uses teaching skills to instruct patient and family rather than doing everything for them

ANS: A An clue to a nontherapeutic staff-patient relationship is concern of other staff members. Allowing the nurse to care for the same patient over time would be therapeutic for the patient and family. Nurses who are able to somewhat withdraw emotionally can protect themselves while providing therapeutic care. Nurses using teaching skills to instruct patient and family will assist in transitioning the child and family to self-care.

An 18-month-old child is seen in the clinic with AOM. Trimethoprim-sulfamethoxazole (Bactrim) is prescribed. Which statement made by the parent indicates a correct understanding of the instructions? a. "I should administer all the prescribed medication." b. "I should continue medication until the symptoms subside." c. "I will immediately stop giving medication if I notice a change in hearing." d. "I will stop giving medication if fever is still present in 24 hours."

ANS: A Antibiotics should be given for their full course to prevent recurrence of infection with resistant bacteria. Symptoms may subside before the full course is given. Hearing loss is a complication of AOM. Antibiotics should continue to be given. Medication may take 24 to 48 hours to make symptoms subside. It should be continued.

The nurse is assessing a child after a cardiac catheterization. Which complication should the nurse be assessing for? a. Cardiac arrhythmia b. Hypostatic pneumonia c. Heart failure d. Rapidly increasing blood pressure

ANS: A Because a catheter is introduced into the heart, a risk exists of catheter-induced dysrhythmias occurring during the procedure. These are usually transient. Hypostatic pneumonia, heart failure, and rapidly increasing blood pressure are not risks usually associated with cardiac catheterization.

A nurse is preparing to assess a 3-year-old child. What communication technique should the nurse use for this child? a. Focus communication on child. b. Explain experiences of others to child. c. Use easy analogies when possible. d. Assure child that communication is private.

ANS: A Because children of this age are able to see things only in terms of themselves, the best approach is to focus communication directly on them. Children should be provided with information about what they can do and how they will feel. With children who are egocentric, experiences of others, analogies, and assurances that the communication is private will not be effective because the child is not capable of understanding.

A school-age child had an upper respiratory tract infection for several days and then began having a persistent dry, hacking cough that was worse at night. The cough has become productive in the past 24 hours. This is most suggestive of which diagnosis? a. Bronchitis b. Bronchiolitis c. Viral-induced asthma d. Acute spasmodic laryngitis

ANS: A Bronchitis is characterized by these symptoms and occurs in children older than 6 years. Bronchiolitis is rare in children older than 2 years. Asthma is a chronic inflammation of the airways that may be exacerbated by a virus. Acute spasmodic laryngitis occurs in children between 3 months and 3 years of age.

A child with cystic fibrosis (CF) receives aerosolized bronchodilator medication. When should this medication be administered? a. Before chest physiotherapy (CPT) b. After CPT c. Before receiving 100% oxygen d. After receiving 100% oxygen

ANS: A Bronchodilators should be given before CPT to open bronchi and make expectoration easier. Aerosolized bronchodilator medications are not helpful when used after CPT. Oxygen administration is necessary only in acute episodes with caution because of chronic carbon dioxide retention.

Parents of a child with cystic fibrosis ask the nurse about genetic implications of the disorder. Which statement, made by the nurse, expresses accurately the genetic implications? a. If it is present in a child, both parents are carriers of this defective gene. b. It is inherited as an autosomal dominant trait. c. It is a genetic defect found primarily in non-Caucasian population groups. d. There is a 50% chance that siblings of an affected child also will be affected.

ANS: A CF is an autosomal recessive gene inherited from both parents and is inherited as an autosomal recessive, not autosomal dominant, trait. CF is found primarily in Caucasian populations. An autosomal recessive inheritance pattern means that there is a 25% chance a sibling will be infected but a 50% chance a sibling will be a carrier.

Samantha, age 5 years, tells the nurse that she "needs a Band-Aid" where she had an injection. Which is the best nursing action? a. Apply a Band-Aid. b. Ask her why she wants a Band-Aid. c. Explain why a Band-Aid is not needed. d. Show her that the bleeding has already stopped.

ANS: A Children at this age group still fear that their insides may leak out at the injection site. Provide the Band-Aid. No explanation should be required. The nurse should be prepared to apply a small Band-Aid after the injection.

Which is an effective strategy to reduce the stress of burn dressing procedures? a. Give the child as many choices as possible. b. Reassure the child that dressing changes are not painful. c. Explain to the child why analgesics cannot be used. d. Encourage the child to master stress with controlled passivity.

ANS: A Children who understand the procedure and have some perceived control demonstrate less maladaptive behavior. They respond well to participating in decisions and should be given as many choices as possible. The dressing change procedure is painful and stressful. Misinformation should not be given to the child. Analgesia and sedation can and should be used. Encouraging the child to master stress with controlled passivity is not a positive coping strategy.

In providing nourishment for a child with cystic fibrosis (CF), which factor should the nurse keep in mind? a. Diet should be high in carbohydrates and protein b. Diet should be high in easily digested carbohydrates and fats c. Most fruits and vegetables are not well tolerated. d. Fats and proteins must be greatly curtailed.

ANS: A Children with CF require a well-balanced, high-protein, high-calorie diet because of impaired intestinal absorption. Enzyme supplementation helps digest foods; other modifications are not necessary. A well-balanced diet containing fruits and vegetables is important. Fats and proteins are a necessary part of a well-balanced diet.

Which is the leading cause of death in infants younger than 1 year? a. Congenital anomalies b. Sudden infant death syndrome c. Respiratory distress syndrome d. Bacterial sepsis of the newborn

ANS: A Congenital anomalies account for 20.1% of deaths in infants younger than 1 year. Sudden infant death syndrome accounts for 8.2% of deaths in this age group. Respiratory distress syndrome accounts for 3.4% of deaths in this age group. Infections specific to the perinatal period account for 2.7% of deaths in this age group.

Which explains why cool-mist vaporizers rather than steam vaporizers are recommended in home treatment of childhood respiratory tract infections? a. They are safer. b. They are less expensive. c. Respiratory secretions are dried. d. A more comfortable environment is produced.

ANS: A Cool-mist vaporizers are safer than steam vaporizers, and little evidence exists to show any advantages to steam. The cost of cool-mist and steam vaporizers is comparable. Steam loosens secretions, not dries them. Both may promote a more comfortable environment, but cool-mist vaporizers present decreased risk for burns and growth of organisms.

A nurse is charting that a hospitalized child has labored breathing. Which describes labored breathing? a. Dyspnea b. Tachypnea c. Hypopnea d. Orthopnea

ANS: A Dyspnea is labored breathing. Tachypnea is rapid breathing. Hypopnea is breathing that is too shallow. Orthopnea is difficulty breathing except in an upright position.

An infant's parents ask the nurse about preventing OM. Which should be recommended? a. Avoid tobacco smoke. b. Use nasal decongestant. c. Avoid children with OM. d. Bottle-feed or breastfeed in a supine position.

ANS: A Eliminating tobacco smoke from the child's environment is essential for preventing OM and other common childhood illnesses. Nasal decongestants are not useful in preventing OM. Children with uncomplicated OM are not contagious unless they show other upper respiratory tract infection (URI) symptoms. Children should be fed in an upright position to prevent OM.

Matthew, age 18 months, has just been admitted with croup. His parent is tearful and tells the nurse, "This is all my fault. I should have taken him to the doctor sooner so he wouldn't have to be here." Which is appropriate in the care plan for this parent who is experiencing guilt? a. Clarify misconception about the illness. b. Explain to the parent that the illness is not serious. c. Encourage the parent to maintain a sense of control. d. Assess further why the parent has excessive guilt feelings.

ANS: A Guilt is a common response of parents when a child is hospitalized. They may blame themselves for the child's illness or for not recognizing it soon enough. The nurse should clarify the nature of the problem and reassure parents that the child is being cared for. Croup is a potentially serious illness. The nurse should not minimize the parent's feelings. It would be difficult for the parent to maintain a sense of control while the child is seriously ill. No further assessment is indicated at this time; guilt is a common response for parents.

The nurse has just started assessing a young child who is febrile and appears very ill. There is hyperextension of the child's head (opisthotonos) with pain on flexion. Which is the most appropriate action? a. Refer for immediate medical evaluation. b. Continue assessment to determine cause of neck pain. c. Ask parent when neck was injured. d. Record "head lag" on assessment record, and continue assessment of child.

ANS: A Hyperextension of the child's head with pain on flexion is indicative of meningeal irritation and needs immediate evaluation; it is not descriptive of head lag. The pain is indicative of meningeal irritation. No indication of injury is present.

The nurse is teaching a group of new parents about the experience of role transition. Which statement by a parent would indicate a correct understanding of the teaching? a. "My marital relationship can have a positive or negative effect on the role transition." b. "If an infant has special care needs, the parents' sense of confidence in their new role is strengthened." c. "Young parents can adjust to the new role easier than older parents." d. "A parent's previous experience with children makes the role transition more difficult."

ANS: A If parents are supportive of each other, they can serve as positive influences on establishing satisfying parental roles. When marital tensions alter caregiving routines and interfere with the enjoyment of the infant, then the marital relationship has a negative effect. Infants with special care needs can be a significant source of added stress. Older parents are usually more able to cope with the greater financial responsibilities, changes in sleeping habits, and reduced time for each other and other children. Parents who have previous experience with parenting appear more relaxed, have less conflict in disciplinary relationships, and are more aware of normal growth and development.

A nurse is conducting an in-service on asthma. Which statement is the most descriptive of bronchial asthma? a. There is heightened airway reactivity. b. There is decreased resistance in the airway. c. The single cause of asthma is an allergic hypersensitivity. d. It is inherited.

ANS: A In bronchial asthma, spasm of the smooth muscle of the bronchi and bronchioles causes constriction, producing impaired respiratory function. In bronchial asthma, there is increased resistance in the airway. There are multiple causes of asthma, including allergens, irritants, exercise, cold air, infections, medications, medical conditions, and endocrine factors. Atopy or development of an immunoglobulin E (IgE)-mediated response is inherited but is not the only cause of asthma.

A 3-year-old girl was adopted immediately after birth. The parents have just asked the nurse how they should tell the child that she is adopted. Which guidelines concerning adoption should the nurse use in planning a response? a. Telling the child is an important aspect of their parental responsibilities. b. The best time to tell the child is between ages 7 and 10 years. c. It is not necessary to tell the child who was adopted so young. d. It is best to wait until the child asks about it.

ANS: A It is important for the parents not to withhold information about the adoption from the child. It is an essential component of the child's identity. There is no recommended best time to tell children. It is believed that children should be told young enough so they do not remember a time when they did not know. It should be done before the children enter school to keep third parties from telling the children before the parents have had the opportunity.

Which approach would be best to use to ensure a positive response from a toddler? a. Assume an eye-level position and talk quietly. b. Call the toddler's name while picking him or her up. c. Call the toddler's name and say, "I'm your nurse." d. Stand by the toddler, addressing him or her by name.

ANS: A It is important that the nurse assume a position at the child's level when communicating with the child. By speaking quietly and focusing on the child, the nurse should be able to obtain a positive response. The nurse should engage the child and inform the toddler what is going to occur. If the nurse picks up the child without explanation, the child is most likely going to become upset. The toddler may not understand the meaning of the phrase, "I'm your nurse." If a positive response is desired, the nurse should assume the child's level when speaking if possible.

A nurse is planning a class on accident prevention for parents of toddlers. Which safety topic is the priority for this class? a. Appropriate use of car seat restraints b. Safety crossing the street c. Helmet use when riding a bicycle d. Poison control numbers

ANS: A Motor vehicle accidents (MVAs) continue to be the most common cause of death in children older than 1 year, therefore the priority topic is appropriate use of car seat restraints. Safety crossing the street and bicycle helmet use are topics that should be included for preschool parents but are not priorities for parents of toddlers. Information about poison control is important for parents of toddlers and would be a safety topic to include but is not the priority over appropriate use of car seat restraints.

The nurse must assess a 10-month-old infant. The infant is sitting on the father's lap and appears to be afraid of the nurse and of what might happen next. Which initial action by the nurse would be most appropriate? a. Initiate a game of peek-a-boo. b. Ask father to place the infant on the examination table. c. Undress the infant while he is still sitting on his father's lap. d. Talk softly to the infant while taking him from his father.

ANS: A Peek-a-boo is an excellent means of initiating communication with infants while maintaining a safe, nonthreatening distance. The child will most likely become upset if separated from his father. As much of the assessment as possible should be done on the father's lap. The nurse should have the father undress the child as needed for the examination.

When assessing a family, the nurse determines that the parents exert little or no control over their children. What is this style of parenting called? a. Permissive b. Dictatorial c. Democratic d. Authoritarian

ANS: A Permissive parents avoid imposing their own standards of conduct and allow their children to regulate their own activity as much as possible. The parents exert little or no control over their children's actions. Dictatorial or authoritarian parents attempt to control their children's behavior and attitudes through unquestioned mandates. They establish rules and regulations or standards of conduct that they expect to be followed rigidly and unquestioningly. Democratic parents combine permissive and dictatorial styles. They direct their children's behavior and attitudes by emphasizing the reasons for rules and negatively reinforcing deviations. They respect the child's individual nature.

The nurse is conducting a staff in-service on childhood-acquired heart diseases. Which is a major clinical manifestation of rheumatic fever? a. Polyarthritis b. Osler nodes c. Janeway spots d. Splinter hemorrhages of distal third of nails

ANS: A Polyarthritis, which is swollen, hot, red, and painful joints, is a major clinical manifestation of rheumatic fever. The affected joints will change every 1 to 2 days. Primarily the large joints are affected. Osler nodes, Janeway spots, and splinter hemorrhages are characteristic of infective endocarditis.

The nurse has determined the rate of both the child's radial pulse and heart. What is the normal finding when comparing the two rates? a. Are the same b. Differ, with heart rate faster c. Differ, with radial pulse faster d. Differ, depending on quality and intensity

ANS: A Pulses are the fluid wave through the blood vessel as a result of each heartbeat. Therefore, they should be the same.

A previously "potty-trained" 30-month-old child has reverted to wearing diapers while hospitalized. The nurse should reassure the parents that this is normal because of which reason? a. Regression is seen during hospitalization. b. Developmental delays occur because of the hospitalization. c. The child is experiencing urinary urgency because of hospitalization. d. The child was too young to be "potty-trained."

ANS: A Regression is expected and normal for all age groups when hospitalized. Nurses should assure the parents this is temporary and the child will return to the previously mastered developmental milestone when back home. This does not indicate a developmental delay. The child should not be experiencing urinary urgency because of hospitalization and this would not be normal. Successful "potty-training" can be started at 2 years of age if the child is ready.

Latasha, age 8 years, is being admitted to the hospital from the emergency department with an injury from falling off her bicycle. Which will help her most in her adjustment to the hospital? a. Explain hospital schedules to her, such as mealtimes. b. Use terms such as "honey" and "dear" to show a caring attitude. c. Explain when parents can visit and why siblings cannot come to see her. d. Orient her parents, because she is young, to her room and hospital facility.

ANS: A School-age children need to have control of their environment. The nurse should offer explanations or prepare the child for those experiences that are unavailable. The nurse should refer to the child by the preferred name. Explaining when parents can visit and why siblings cannot come to see her is telling the child all of the limitations, not helping her adjust to the hospital. At the age of 8 years, the child should be oriented to the environment along with the parents.

The nurse is conducting a staff in-service on congenital heart defects. Which structural defect constitutes tetralogy of Fallot? a. Pulmonic stenosis, ventricular septal defect, overriding aorta, right ventricular hypertrophy b. Aortic stenosis, ventricular septal defect, overriding aorta, right ventricular hypertrophy c. Aortic stenosis, atrial septal defect, overriding aorta, left ventricular hypertrophy d. Pulmonic stenosis, ventricular septal defect, aortic hypertrophy, left ventricular hypertrophy

ANS: A Tetralogy of Fallot has these four characteristics: pulmonic stenosis, ventricular septal defect, overriding aorta, and right ventricular hypertrophy. There is pulmonic stenosis but not atrial stenosis in tetralogy of Fallot. Right ventricular hypertrophy, not left ventricular hypertrophy, is present in tetralogy of Fallot. Tetralogy of Fallot has right ventricular hypertrophy, not left ventricular hypertrophy, and an atrial septal defect, not aortic hypertrophy.

A 10-year-old girl needs to have another intravenous (IV) line started. She keeps telling the nurse, "Wait a minute" and "I'm not ready." The nurse should recognize this as which description? a. This is normal behavior for a school-age child. b. The behavior is not seen past the preschool years. c. The child thinks the nurse is punishing her. d. The child has successfully manipulated the nurse in the past.

ANS: A The 10-year-old girl is attempting to maintain control. The nurse should provide the girl with structured choices about when the IV will be inserted. Telling the nurse "Wait a minute" and "I'm not ready" can be characteristic behavior when an individual needs to maintain some control over a situation.

Which would be best for the nurse to use when determining the temperature of a preterm infant under a radiant heater? a. Axillary sensor b. Tympanic membrane sensor c. Rectal mercury glass thermometer d. Rectal electronic thermometer

ANS: A The axillary sensor measures the infrared heat energy radiating from the axilla. It can be used on wet skin, in incubators, or under radiant warmers. Ear thermometry does not show sufficient correlation with established methods of measurement. It should not be used when body temperature must be assessed with precision. Mercury thermometers should never be used. The release of mercury, should the thermometer be broken, can cause harmful vapors. Rectal temperatures should be avoided unless no other suitable way exists for the temperature to be measured.

Parents of a 3-year-old child with congenital heart disease are afraid to let their child play with other children because of possible overexertion. The nurse's reply should be based on which statement? a. The child needs opportunities to play with peers. b. The child needs to understand that peers' activities are too strenuous. c. Parents can meet all of the child's needs. d. Constant parental supervision is needed to avoid overexertion.

ANS: A The child needs opportunities for social development. Children usually limit their activities if allowed to set their own pace. The child will limit activities as necessary. Parents must be encouraged to seek appropriate social activities for the child, especially before kindergarten. The child needs to have activities that foster independence. The child will be able to regulate activities.

A school-age child has been admitted with an acute asthma episode. The child is receiving oxygen by nasal prongs at 2 liters. How often should the nurse plan to monitor the child's pulse oximetry status? a. Continuous b. Every 30 minutes c. Every hour d. Every 2 hours

ANS: A The child on supplemental oxygen requires intermittent or continuous oxygenation monitoring, depending on severity of respiratory compromise and initial oxygenation status. The child in status asthmaticus should be placed on continuous cardiorespiratory (including blood pressure) and pulse oximetry monitoring.

A nurse is teaching nursing students about clinical manifestations of cystic fibrosis (CF). Which is/are the earliest recognizable clinical manifestation(s) of CF? a. Meconium ileus b. History of poor intestinal absorption c. Foul-smelling, frothy, greasy stools d. Recurrent pneumonia and lung infections

ANS: A The earliest clinical manifestation of CF is a meconium ileus, which is found in about 10% of children with CF. Clinical manifestations include abdominal distention, vomiting, failure to pass stools, and rapid development of dehydration. History of malabsorption is a later sign that manifests as failure to thrive. Foul-smelling stools are a later manifestation of CF. Recurrent respiratory tract infections are a later sign of CF.

The nurse is admitting a child with coarctation of the aorta. Which figure depicts this congenital heart defect?

ANS: A The figure that depicts a narrowing of the aortic arch is coarctation of the aorta. It typically occurs past the ductal area but can occur in other areas along the aortic arch. The figure that depicts an opening between the atria is atrial septal defect. The figure that depicts an opening between the ventricles is ventricular septal defect. The figure that depicts an opening from the atrium to the pulmonary artery is patent ductus arteriosus.

The nurse is seeing an adolescent boy and his parents in the clinic for the first time. What should the nurse do first? a. Introduce self. b. Make family comfortable. c. Explain purpose of interview. d. Give assurance of privacy.

ANS: A The first thing that nurses should do is to introduce themselves to the patient and family. Parents and other adults should be addressed with appropriate titles unless they specify a preferred name. During the initial part of the interview, the nurse should include general conversation to help make the family feel at ease. Clarification of the purpose of the interview and the nurse's role is the next thing that should be done. The interview should take place in an environment as free of distraction as possible. In addition, the nurse should clarify which information will be shared with other members of the health care team and any limits to the confidentiality.

The nurse is caring for an infant with congestive heart disease (CHD). The nurse should plan which intervention to decrease cardiac demands? a. Organize nursing activities to allow for uninterrupted sleep. b. Allow the infant to sleep through feedings during the night. c. Wait for the infant to cry to show definite signs of hunger. d. Discourage parents from rocking the infant

ANS: A The infant requires rest and conservation of energy for feeding. Every effort is made to organize nursing activities to allow for uninterrupted periods of sleep. Whenever possible, parents are encouraged to stay with their infant to provide the holding, rocking, and cuddling that help children sleep more soundly. To minimize disturbing the infant, changing bed linens and complete bathing are done only when necessary. Feeding is planned to accommodate the infant's sleep and wake patterns. The child is fed at the first sign of hunger, such as when sucking on fists, rather than waiting until he or she cries for a bottle because the stress of crying exhausts the limited energy supply. Because infants with CHD tire easily and may sleep through feedings, smaller feedings every 3 hours may be helpful.

A nurse is admitting a toddler to the hospital. The parent needs to leave for a brief period. Which figure depicts the reaction the nurse expects from the child?

ANS: A The major stress from middle infancy throughout the preschool years, especially for children ages 6 to 30 months, is separation anxiety, also called anaclitic depression. During the stage of protest, children react aggressively to the separation from the parent. They cry and scream for their parents, refuse the attention of anyone else, and are inconsolable in their grief. When the parent leaves even for a short time this is the expected reaction and the figure that depicts the child not wanting the parent to leave is what the nurse should expect as a reaction from the child. The child sitting alone sadly depicts a child in the despair stage. In this stage depression is evident. The child is much less active, is uninterested in play or food, and withdraws from others. The child sitting on the parent's lap is withdrawn and sad, even in the presence of the parent. The child depicted playing a game is adjusting to the hospitalization with play.

A nurse is caring for four patients; three are toddlers and one is a preschooler. Which represents the major stressor of hospitalization for these four patients? a. Separation anxiety b. Loss of control c. Fear of bodily injury d. Fear of pain

ANS: A The major stressor for children from infancy through the preschool years is separation anxiety, also called anaclitic depression. This is a major stressor of hospitalization. Loss of control, fear of bodily injury, and fear of pain are all stressors associated with hospitalization. However, separation from family is a primary stressor in this age group.

The nurse is preparing to perform a physical assessment on a 10-year-old girl. The nurse gives her the option of her mother either staying in the room or leaving. How should this action be interpreted? a. Appropriate because of child's age b. Appropriate because mother would be uncomfortable making decisions for child c. Inappropriate because of child's age d. Inappropriate because child is same sex as mother

ANS: A The older school-age child should be given the option of having the parent present or not. During the examination, the nurse should respect the child's need for privacy. Although the question was appropriate for the child's age, the mother is responsible for making decisions for the child. It is appropriate because of the child's age. During the examination, the nurse must respect the child's privacy. The child should help determine who is present during the examination.

The nurse is interviewing the mother of an infant. She reports, "I had a difficult delivery, and my baby was born preterm." This information should be recorded under which of the following headings? a. Past history b. Present illness c. Chief complaint d. Review of systems

ANS: A The past history refers to information that relates to previous aspects of the child's health, not to the current problem. The mother's difficult delivery and prematurity are important parts of the past history of an infant. The history of the present illness is a narrative of the chief complaint from its earliest onset through its progression to the present. Unless the chief complaint is directly related to the prematurity, this information is not included in the history of present illness. The chief complaint is the specific reason for the child's visit to the clinic, office, or hospital. It would not include the birth information. The review of systems is a specific review of each body system. It does not include the preterm birth. Sequelae such as pulmonary dysfunction would be included.

The nurse has a 2-year-old boy sit in "tailor" position during palpation for the testes. What is the rationale for this position? a. It prevents cremasteric reflex. b. Undescended testes can be palpated. c. This tests the child for an inguinal hernia. d. The child does not yet have a need for privacy.

ANS: A The tailor position stretches the muscle responsible for the cremasteric reflex. This prevents its contraction, which pulls the testes into the pelvic cavity. Undescended testes cannot be predictably palpated. Inguinal hernias are not detected by this method. This position is used for inhibiting the cremasteric reflex. Privacy should always be provided for children.

Decongestant nose drops are recommended for a 10-month-old infant with an upper respiratory tract infection. Instructions for nose drops should include which action? a. Avoid using for more than 3 days. b. Keep drops to use again for nasal congestion. c. Administer drops until nasal congestion subsides. d. Administer drops after feedings and at bedtime.

ANS: A Vasoconstrictive nose drops such as phenylephrine (Neo-Synephrine) should not be used for more than 3 days to avoid rebound congestion. Drops should be discarded after one illness because they may become contaminated with bacteria. Vasoconstrictive nose drops can have a rebound effect after 3 days of use. Drops administered before feedings are more helpful.

What type of breath sound is normally heard over the entire surface of the lungs except for the upper intrascapular area and the area beneath the manubrium? a. Vesicular b. Bronchial c. Adventitious d. Bronchovesicular

ANS: A Vesicular breath sounds are heard over the entire surface of lungs, with the exception of the upper intrascapular area and the area beneath the manubrium. Bronchial breath sounds are heard only over the trachea near the suprasternal notch. Adventitious breath sounds are not usually heard over the chest. These sounds occur in addition to normal or abnormal breath sounds. Bronchovesicular breath sounds are heard over the manubrium and in the upper intrascapular regions where trachea and bronchi bifurcate.

A nurse is performing an otoscopic exam on a school-age child. Which direction should the nurse pull the pinna for this age of child? a. Up and back b. Down and back c. Straight back d. Straight up

ANS: A With older children, usually those older than 3 years of age, the canal curves downward and forward. Therefore, pull the pinna up and back during otoscopic examinations. In infants, the canal curves upward. Therefore, pull the pinna down and back to straighten the canal. Pulling the pinna straight back or straight up will not open the inner ear canal.

A nurse is performing an assessment on a school-age child. Which findings suggest the child is getting an excess of vitamin A? (Select all that apply.) a. Delayed sexual development b. Edema c. Pruritus d. Jaundice e. Paresthesia

ANS: A, C, D Excess vitamin A can cause delayed sexual development, pruritus, and jaundice. Edema is seen with excess sodium. Paresthesia occurs with excess riboflavin.

The nurse is caring for a 5-year-old child who is scheduled for a tonsillectomy in 2 hours. Which action should the nurse include in the child's postoperative care plan? (Select all that apply.) a. Notify the surgeon if the child swallows frequently. b. Apply a heat collar to the child for pain relief. c. Place the child on the abdomen until fully wake. d. Allow the child to have diluted juice after the procedure. e. Encourage the child to cough frequently.

ANS: A, C, D Frequent swallowing is a sign of bleeding in children after a tonsillectomy. The child should be placed on the abdomen or the side to facilitate drainage. The child can drink diluted juice, cool water, or popsicles after the procedure. An ice collar should be used after surgery. Frequent coughing and nose blowing should be avoided.

A child has just been unexpectedly admitted to the intensive care unit after abdominal surgery. The nursing staff has completed the admission process, and the child's condition is beginning to stabilize. When speaking with the parents, the nurses should expect which stressors to be evident? (Select all that apply.) a. Unfamiliar environment b. Usual day-night routine c. Strange smells d. Provision of privacy e. Inadequate knowledge of condition and routine

ANS: A, C, E Intensive care units, especially when the family is unprepared for the admission, are a strange and unfamiliar place with many pieces of unfamiliar equipment. The sights and sounds are much different from those of a general hospital unit. Also, with the child's condition being more precarious, it may be difficult to keep the parents updated and knowledgeable about what is happening. Lights are usually on around the clock, seriously disrupting the diurnal rhythm. There is usually little privacy available for families in intensive care units.

A child is being discharged from an ambulatory care center after an inguinal hernia repair. Which discharge interventions should the nurse implement? (Select all that apply.) a. Discuss dietary restrictions. b. Hold any analgesic medications until the child is home. c. Send a pain scale home with the family. d. Suggest the parents fill the prescriptions on the way home. e. Discuss complications that may occur.

ANS: A, C, E The discharge interventions a nurse should implement when a child is being discharged from an ambulatory care center should include dietary restrictions, being very specific and giving examples of "clear fluids" or what is meant by a "full liquid diet." The nurse should give specific information on pain control and send a pain scale home with the family. All complications that may occur after an inguinal hernia repair should be discussed with the parents. The pain medication, as prescribed, should be given before the child leaves the building and prescriptions should be filled and given to the family before discharge.

Which of the following data would be included in a health history? (Select all that apply.) a. Review of systems b. Physical assessment c. Sexual history d. Growth measurements e. Nutritional assessment f. Family medical history

ANS: A, C, E, F The review of systems, sexual history, nutritional assessment, and family medical history are part of the health history. Physical assessment and growth measurements are components of the physical examination.

An infant has developed staphylococcal pneumonia. Nursing care of the child with pneumonia includes which interventions? (Select all that apply.) a. Cluster care to conserve energy b. Round-the-clock administration of antitussive agents c. Strict intake and output to avoid congestive heart failure d. Administration of antibiotics

ANS: A, D Antibiotics are indicated for a bacterial pneumonia. Often the child will have decreased pulmonary reserve, and the clustering of care is essential. Antitussive agents are used sparingly. It is desirable for the child to cough up some of the secretions. Fluids are essential to kept secretions as liquefied as possible.

A child is being admitted to the intensive care unit (ICU) and the parents are with the child. Which creates stressors for children and parents in ICUs? (Select all that apply.) a. Equipment noise b. Privacy c. Caring behavior by the nurse d. Unfamiliar smells e. Sleep deprivation

ANS: A, D, E The ICU can create physical and environmental stressors for children and their families. Equipment noise (monitors, suction equipment, telephones, computers), unfamiliar smells (alcohol, adhesive remover, body odors), and sleep deprivation all are stressors found in the ICU. Privacy as opposed to no privacy and a caring nurse as opposed to unkind or thoughtless comments from staff help reduce the stressors of the ICU.

A nurse is planning to use an interpreter during a health history interview of a non-English speaking patient and family. Which nursing care guidelines should the nurse include when using an interpreter? (Select all that apply.) a. Elicit one answer at a time. b. Interrupt the interpreter if the response from the family is lengthy. c. Comments to the interpreter about the family should be made in English. d. Arrange for the family to speak with the same interpreter, if possible. e. Introduce the interpreter to the family.

ANS: A, D, E When using an interpreter, the nurse should pose questions to elicit only one answer at a time, such as: "Do you have pain?" rather than "Do you have any pain, tiredness, or loss of appetite?" Refrain from interrupting family members and the interpreter while they are conversing. Introduce the interpreter to family and allow some time before the interview for them to become acquainted. Refrain from interrupting family members and the interpreter while they are conversing. Avoid commenting to the interpreter about family members because they may understand some English.

The nurse is meeting a 5-year-old child for the first time and would like the child to cooperate during a dressing change. The nurse decides to do a simple magic trick using gauze. How should this action be interpreted? a. Inappropriate, because of child's age b. A way to establish rapport c. Too distracting, when cooperation is important d. Acceptable, if there is adequate time

ANS: B A magic trick or other simple game may help alleviate anxiety for a 5-year-old. It is an excellent method to build rapport and facilitate cooperation during a procedure. Magic tricks appeal to the natural curiosity of young children. The nurse should establish rapport with the child. Failure to do so may cause the procedure to take longer and be more traumatic.

When the nurse interviews an adolescent, which is especially important? a. Focus the discussion on the peer group. b. Allow an opportunity to express feelings. c. Emphasize that confidentiality will always be maintained. d. Use the same type of language as the adolescent.

ANS: B Adolescents, like all children, need an opportunity to express their feelings. Often they will interject feelings into their words. The nurse must be alert to the words and feelings expressed. Although the peer group is important to this age group, the focus of the interview should be on the adolescent. The nurse should clarify which information will be shared with other members of the health care team and any limits to confidentiality. The nurse should maintain a professional relationship with adolescents. To avoid misinterpretation of words and phrases that the adolescent may use, the nurse should clarify terms frequently.

The nurse is caring for a child with carbon monoxide poisoning associated with smoke inhalation. Which is essential in this child's care? a. Monitor pulse oximetry. b. Monitor arterial blood gases. c. Administer oxygen if respiratory distress develops. d. Administer oxygen if child's lips become bright, cherry red.

ANS: B Arterial blood gases are the best way to monitor carbon monoxide poisoning. Pulse oximetry is contraindicated in the case of carbon monoxide poisoning because the PaO2 may be normal. The child should receive 100% oxygen as quickly as possible, not only if respiratory distress or other symptoms develop.

The nurse is taking a sexual history on an adolescent girl. Which is the best way to determine whether she is sexually active? a. Ask her, "Are you sexually active?" b. Ask her, "Are you having sex with anyone?" c. Ask her, "Are you having sex with a boyfriend?" d. Ask both the girl and her parent whether she is sexually active.

ANS: B Asking the adolescent girl whether she is having sex with anyone is a direct question that is well understood. The phrase sexually active is broadly defined and may not provide specific information to the nurse to provide necessary care. The word anyone is preferred to using gender-specific terms such as boyfriend or girlfriend. Because homosexual experimentation may occur, it is preferable to use gender-neutral terms. Questioning about sexual activity should occur when the adolescent is alone.

Which is most descriptive of clinical reasoning? a. A simple developmental process b. Purposeful and goal-directed c. Based on deliberate and irrational thought d. Assists individuals in guessing what is most appropriate

ANS: B Clinical reasoning is a complex, developmental process based on rational and deliberate thought. Clinical reasoning is not a developmental process. Clinical reasoning is based on rational and deliberate thought. Clinical reasoning is not a guessing process.

Which term best describes a group of people who share a set of values, beliefs, practices, social relationships, law, politics, economics, and norms of behavior? a. Race b. Culture c. Ethnicity d. Social group

ANS: B Culture is a pattern of assumptions, beliefs, and practices that unconsciously frames or guides the outlook and decisions of a group of people. A culture is composed of individuals who share a set of values, beliefs, and practices that serve as a frame of reference for individual perceptions and judgments. Race is defined as a division of humankind possessing traits that are transmissible by descent and are sufficient to characterize it as a distinct human type. Ethnicity is an affiliation of a set of persons who share a unique cultural, social, and linguistic heritage. A social group consists of systems of roles carried out in groups. Examples of primary social groups include the family and peer groups.

As part of the treatment for heart failure, the child takes the diuretic furosemide (Lasix). As part of teaching home care, the nurse encourages the family to give the child foods such as bananas, oranges, and leafy vegetables. These foods are recommended because they are high in which nutrient? a. Chlorides b. Potassium c. Sodium d. Vitamins

ANS: B Diuretics that work on the proximal and distal renal tubules contribute to increased losses of potassium. The child's diet should be supplemented with this electrolyte. With this type of diuretic, potassium must be monitored and supplemented as needed.

Which consideration is the most important in managing tuberculosis (TB) in children? a. Skin testing annually b. Pharmacotherapy c. Adequate nutrition d. Adequate hydration

ANS: B Drug therapy for TB includes isoniazid, rifampin, and pyrazinamide daily for 2 months and two or three times a week for the remaining 4 months. Pharmacotherapy is the most important intervention for TB.

When discussing discipline with the mother of a 4-year-old child, the nurse should include which instruction? a. Children as young as 4 years old rarely need to be punished. b. Parental control should be consistent. c. Withdrawal of love and approval is effective at this age. d. One should expect rules to be followed rigidly and unquestioningly.

ANS: B For effective discipline, parents must be consistent and must follow through with agreed-on actions. Realistic goals should be set for this age group. Parents should structure the environment to prevent unnecessary difficulties. Requests for behavior change should be phrased in a positive manner to provide direction for the child. Withdrawal of love and approval is never appropriate or effective. Discipline strategies should be appropriate to the child's age, temperament, and severity of the misbehavior. Following rules rigidly and unquestioningly is beyond the developmental capabilities of a 4-year-old

Which is an important consideration for the nurse when changing dressings and applying topical medication to a child's abdomen and leg burns? a. Apply topical medication with clean hands. b. Wash hands and forearms before and after dressing change. c. If dressings adhere to the wound, soak in hot water before removal. d. Apply dressing so that movement is limited during the healing process.

ANS: B Frequent hand and forearm washing is the single most important element of the infection-control program. Topical medications should be applied with a tongue blade or gloved hand. Dressings that have adhered to the wound can be removed with tepid water or normal saline. Dressings are applied with sufficient tension to remain in place but not so tightly as to impair circulation or limit motion.

An 8-year-old child is receiving digoxin (Lanoxin). The nurse should notify the practitioner and withhold the medication if the apical pulse is less than _____ beats/min. a. 60 b. 70 c. 90 d. 100

ANS: B If a 1-minute apical pulse is less than 70 beats/min for an older child, the digoxin is withheld; 60 beats/min is the cut-off for holding the digoxin dose in an adult. A pulse below 90 to 110 beats/min is the determination for not giving a digoxin dose to infants and young children.

The nurse is assessing a child with acute epiglottitis. Examining the child's throat by using a tongue depressor might precipitate which symptom or condition? a. Inspiratory stridor b. Complete obstruction c. Sore throat d. Respiratory tract infection

ANS: B If a child has acute epiglottitis, examination of the throat may cause complete obstruction and should be performed only when immediate intubation can take place. Stridor is aggravated when a child with epiglottitis is supine. Sore throat and pain on swallowing are early signs of epiglottitis. Epiglottitis is caused by H. influenzae in the respiratory tract.

Cardiopulmonary resuscitation (CPR) is begun on a toddler. Which pulse is usually palpated because it is the most central and accessible? a. Radial b. Carotid c. Femoral d. Brachial

ANS: B In a toddler, the carotid pulse is palpated. The radial pulse is not considered a central pulse. The femoral pulse is not the most central and accessible. Brachial pulse is felt in infants younger than 1 year.

The nurse is caring for a child with persistent hypoxia secondary to a cardiac defect. The nurse recognizes that a risk exists of cerebrovascular accidents (strokes). Which is an important objective to decrease this risk? a. Minimize seizures b. Prevent dehydration c. Promote cardiac output d. Reduce energy expenditure

ANS: B In children with persistent hypoxia, polycythemia develops. Dehydration must be prevented in hypoxemic children because it potentiates the risk of strokes. Minimizing seizures, promoting cardiac output, and reducing energy expenditure will not reduce the risk of cerebrovascular accidents.

The nurse is preparing an in-service education to staff about atraumatic care for pediatric patients. Which intervention should the nurse include? a. Prepare the child for separation from parents during hospitalization by reviewing a video. b. Prepare the child before any unfamiliar treatment or procedure by demonstrating on a stuffed animal. c. Help the child accept the loss of control associated with hospitalization. d. Help the child accept pain that is connected with a treatment or procedure.

ANS: B Preparing the child for any unfamiliar treatments, controlling pain, allowing privacy, providing play activities for expression of fear and aggression, providing choices, and respecting cultural differences are components of atraumatic care. In the provision of atraumatic care, the separation of child from parents during hospitalization is minimized. The nurse should promote a sense of control for the child. Preventing and minimizing bodily injury and pain are major components of atraumatic care.

Amy, age 6 years, needs to be hospitalized again because of a chronic illness. The clinic nurse overhears her school-age siblings tell her, "We are sick of Mom always sitting with you in the hospital and playing with you. It isn't fair that you get everything and we have to stay with the neighbors." Which is the nurse's best assessment of this situation? a. The siblings are immature and probably spoiled. b. Jealousy and resentment are common reactions to the illness or hospitalization of a sibling. c. Family has ineffective coping mechanisms to deal with chronic illness. d. The siblings need to better understand their sister's illness and needs.

ANS: B Siblings experience loneliness, fear, and worry, as well as anger, resentment, jealousy, and guilt. The siblings experience stress equal to that of the hospitalized child. There is no evidence that the family has maladaptive coping mechanisms.

Which should the nurse consider when preparing a school-age child and the family for heart surgery? a. Unfamiliar equipment should not be shown. b. Let the child hear the sounds of an ECG monitor. c. Avoid mentioning postoperative discomfort and interventions. d. Explain that an endotracheal tube will not be needed if the surgery goes well.

ANS: B The child and family should be exposed to the sights and sounds of the intensive care unit (ICU). All positive, nonfrightening aspects of the environment are emphasized. The child should be shown unfamiliar equipment and its use demonstrated on a doll. Carefully prepare the child for the postoperative experience, including intravenous (IV) lines, incision, and endotracheal tube.

The nurse is preparing an adolescent for discharge after a cardiac catheterization. Which statement by the adolescent would indicate a need for further teaching? a. "I should avoid tub baths but may shower." b. "I have to stay on strict bed rest for 3 days." c. "I should remove the pressure dressing the day after the procedure." d. "I may attend school but should avoid exercise for several days."

ANS: B The child does not need to be on strict bed rest for 3 days. Showers are recommended; children should avoid a tub bath. The pressure dressing is removed the day after the catheterization and replaced by an adhesive bandage to keep the area clean. Strenuous activity must be avoided for several days, but the child can return to school.

An 8-month-old infant has a hypercyanotic spell while blood is being drawn. What is the priority nursing action? a. Assess for neurologic defects b. Place the child in the knee-chest position c. Begin cardiopulmonary resuscitation d. Prepare family for imminent death

ANS: B The first action is to place the infant in the knee-chest position. Blow-by oxygen may be indicated. Neurologic defects are unlikely. The child should be assessed for airway, breathing, and circulation. Often, calming the child and administering oxygen and morphine can alleviate the hypercyanotic spell.

A child is diagnosed with influenza. Management includes which recommendation? a. Clear liquid diet for hydration b. Aspirin to control fever c. Oseltamivie (Tamiflu) d. Antibiotics to prevent bacterial infection

ANS: C Oseltamivie (Tamiflu) may reduce symptoms related to influenza A if administered within 24 to 48 hours of onset. A clear liquid diet is not necessary for influenza, but maintaining hydration is important. Aspirin is not recommended in children because of increased risk of Reye syndrome. Acetaminophen or ibuprofen is a better choice. Preventive antibiotics are not indicated for influenza unless there is evidence of a secondary bacterial infection.

A nurse in the emergency department is assessing a 5-year-old child with symptoms of pneumonia and a fever of 102° F. Which intervention can the nurse implement to promote a sense of control for the child? a. None; this is an emergency and the child should not participate in care. b. Allow the child to hold the digital thermometer while taking the child's blood pressure. c. Ask the child if it is OK to take a temperature in the ear. d. Have parents wait in the waiting room.

ANS: B The nurse should allow the child to hold the digital thermometer while taking the child's blood pressure. Unless an emergency is life threatening, children need to participate in their care to maintain a sense of control. Because emergency departments are frequently hectic, there is a tendency to rush through procedures to save time. However, the extra few minutes needed to allow children to participate may save many more minutes of useless resistance and uncooperativeness during subsequent procedures. The child may not give permission, if asked, for a procedure that is necessary to be performed. It is better to give choices such as, "Which ear do you want me to do your temperature in?" instead of, "Can I take your temperature?" Parents should remain with their child to help with decreasing the child's anxiety.

Which is the leading cause of death after heart transplantation? a. Infection b. Rejection c. Cardiomyopathy d. Heart failure

ANS: B The posttransplant course is complex. The leading cause of death after cardiac transplantation is rejection. Infection is a continued risk secondary to the immunosuppression necessary to prevent rejection. Cardiomyopathy is one of the indications for cardiac transplant. Heart failure is not a leading cause of death.

What is an important consideration for the nurse who is communicating with a very young child? a. Speak loudly, clearly, and directly. b. Use transition objects, such as a doll. c. Disguise own feelings, attitudes, and anxiety. d. Initiate contact with child when parent is not present.

ANS: B Using a transition object allows the young child an opportunity to evaluate an unfamiliar person (the nurse). This will facilitate communication with a child this age. Speaking in this manner will tend to increase anxiety in very young children. The nurse must be honest with the child. Attempts at deception will lead to a lack of trust. Whenever possible, the parent should be present for interactions with young children.

Which communication technique should the nurse avoid when interviewing children and their families? a. Using silence b. Using clichés c. Directing the focus d. Defining the problem

ANS: B Using stereotyped comments or clichés can block effective communication, and this technique should be avoided. After use of such trite phrases, parents will often not respond. Silence can be an effective interviewing tool. Silence permits the interviewee to sort out thoughts and feelings and search for responses to questions. To be effective, the nurse must be able to direct the focus of the interview while allowing maximal freedom of expression. By using open-ended questions, along with guiding questions, the nurse can obtain the necessary information and maintain the relationship with the family. The nurse and parent must collaborate and define the problem that will be the focus of the nursing intervention.

A nurse is admitting an infant with asthma. What usually triggers asthma in infants? a. Medications b. A viral infection c. Exposure to cold air d. Allergy to dust or dust mites

ANS: B Viral illnesses cause inflammation that causes increased airway reactivity in asthma. Medications such as aspirin, nonsteroidal anti-inflammatory drugs (NSAIDs), and antibiotics may aggravate asthma, but not frequently in infants. Exposure to cold air may exacerbate already existing asthma. Allergy is associated with asthma, but 20% to 40% of children with asthma have no evidence of allergic disease.

The nurse is teaching parents about signs of digoxin (Lanoxin) toxicity. Which is a common sign of digoxin toxicity? a. Seizures b. Vomiting c. Bradypnea d. Tachycardia

ANS: B Vomiting is a common sign of digoxin toxicity. Seizures are not associated with digoxin toxicity. The child will have a slower heart rate, not respiratory rate. The heart rate will be slower, not faster.

β-Adrenergic agonists and methylxanthines are often prescribed for a child with an asthma attack. Which describes their action? a. Liquefy secretions b. Dilate the bronchioles c. Reduce inflammation of the lungs d. Reduce infection

ANS: B β-Adrenergic agonists and methylxanthines work to dilate the bronchioles in acute exacerbations. These medications do not liquefy secretions or reduce infection. Corticosteroids and mast cell stabilizers reduce inflammation in the lungs.

A nurse is caring for a school-age child with left unilateral pneumonia and pleural effusion. A chest tube has been inserted to promote continuous closed chest drainage. Which interventions should the nurse implement when caring for this child? (Select all that apply.) a. Positioning child on the right side b. Assessing the chest tube and drainage device for correct settings c. Administering prescribed doses of analgesia d. Clamping the chest tube when child ambulates e. Monitoring for need of supplemental oxygen

ANS: B, C, E Nursing care of the child with a chest tube requires close attention to respiratory status; the chest tube and drainage device used are monitored for proper function (i.e., drainage is not impeded, vacuum setting is correct, tubing is free of kinks, dressing covering chest tube insertion site is intact, water seal is maintained, and chest tube remains in place). Movement in bed and ambulation with a chest tube are encouraged according to the child's respiratory status, but children require frequent doses of analgesia. Supplemental oxygen may be required in the acute phase of the illness and may be administered by nasal cannula, face mask, flow-by, or face tent. The child should be positioned on the left side, not the right. Lying on the affected side if the pneumonia is unilateral ("good lung up") splints the chest on that side and reduces the pleural rubbing that often causes discomfort. The chest tube should never be clamped; this can cause a pneumothorax. The chest tube should be maintained to the underwater seal at all times.

A chest radiograph film is ordered for a child with suspected cardiac problems. The child's parent asks the nurse, "What will the radiograph show about the heart?" What knowledge about the x-ray should the nurse include in the response to the parents? a. Bones of chest but not the heart b. Measurement of electrical potential generated from heart muscle c. Permanent record of heart size and configuration d. Computerized image of heart vessels and tissues

ANS: C A chest radiograph will provide information on the heart size and pulmonary blood-flow patterns. It will provide a baseline for future comparisons. The heart will be visible, as well as the sternum and ribs. Electrocardiography (ECG) measures the electrical potential generated from heart muscle. Echocardiography will produce a computerized image of the heart vessels and tissues by using sound waves.

A child is brought to the emergency department experiencing an anaphylactic reaction to a bee sting. While an airway is being established, the nurse should prepare which medication for immediate administration? a. Diphenhydramine (Benadryl) b. Dobutamine (Dobutarex) c. Epinephrine (Adrenalin) d. Calcium chloride (calcium chloride)

ANS: C After the first priority of establishing an airway, administration of epinephrine is the drug of choice. Diphenhydramine, an antihistamine, is usually not used for severe reactions. Dobutamine and calcium chloride are not appropriate drugs for this type of reaction.

During the first 4 days of hospitalization, Eric, age 18 months, cried inconsolably when his parents left him, and he refused the staff's attention. Now the nurse observes that Eric appears to be "settled in" and unconcerned about seeing his parents. The nurse should interpret this as which statement? a. He has successfully adjusted to the hospital environment. b. He has transferred his trust to the nursing staff. c. He may be experiencing detachment, which is the third stage of separation anxiety. d. Because he is "at home" in the hospital now, seeing his mother frequently will only start the cycle again.

ANS: C Detachment is a behavior manifestation of separation anxiety. Superficially it appears that the child has adjusted to the loss. Detachment is a sign of resignation, not contentment. Parents should be encouraged to be with their child. If parents restrict visits, they may begin a pattern of misunderstanding the child's cues and not meeting his needs.

Which drug is considered the most useful in treating childhood cardiac arrest? a. Bretylium tosylate (Bretylium) b. Lidocaine hydrochloride (Lidocaine) c. Epinephrine hydrochloride (Adrenaline) d. Naloxone (Narcan)

ANS: C Epinephrine works on alpha and beta receptors in the heart and is the most useful drug in childhood cardiac arrest. Bretylium is no longer used in pediatric cardiac arrest management. Lidocaine is used for ventricular arrhythmias only. Naloxone is useful only to reverse effects of opioids.

Which family theory explains how families react to stressful events and suggests factors that promote adaptation to these events? a. Interactional theory b. Developmental systems theory c. Family stress theory d. Duvall's developmental theory

ANS: C Family stress theory explains the reaction of families to stressful events. In addition, the theory helps suggest factors that promote adaptation to the stress. Stressors, both positive and negative, are cumulative and affect the family. Adaptation requires a change in family structure or interaction. Interactional theory is not a family theory. Interactions are the basis of general systems theory. Developmental systems theory is an outgrowth of Duvall's theory. The family is described as a small group, a semiclosed system of personalities that interact with the larger cultural system. Changes do not occur in one part of the family without changes in others. Duvall's developmental theory describes eight developmental tasks of the family throughout its life span.

When interviewing the mother of a 3-year-old child, the nurse asks about developmental milestones such as the age of walking without assistance. How should this question be considered? a. Unnecessary information because child is age 3 years b. An important part of the family history c. An important part of the child's past history d. An important part of the child's review of systems

ANS: C Information about the attainment of developmental milestones is important to obtain. It provides data about the child's growth and development that should be included in the past history. Developmental milestones provide important information about the child's physical, social, and neurologic health and should be included in the history for a 3-year-old child. If pertinent, attainment of milestones by siblings would be included in the family history. The review of systems does not include the developmental milestones.

A nurse is teaching an adolescent how to use the peak expiratory flowmeter. The adolescent has understood the teaching if which statement is made? a. "I will record the average of the readings." b. "I should be sitting comfortably when I perform the readings." c. "I will record the readings at the same time every day." d. "I will repeat the routine two times."

ANS: C Instructions for use of a peak flowmeter include standing up straight before performing the reading, recording the highest of the three readings (not the average), measuring the peak expiratory flow rate (PEFR) close to the same time each day, and repeating the entire routine three times, waiting 30 seconds between each routine.

Which is an important nursing consideration when chest tubes will be removed from a child? a. Explain that it is not painful. b. Explain that only a Band-Aid will be needed. c. Administer analgesics before the procedure. d. Expect bright red drainage for several hours after removal.

ANS: C It is appropriate to prepare the child for the removal of chest tubes with analgesics. Short-acting medications can be used that are administered through an existing IV line. A sharp, momentary pain is felt. This should not be misrepresented to the child. A petroleum gauze, air-tight dressing will be needed, but it is not a pain-free procedure. Little or no drainage should be found on removal.

The nurse observes yellow staining in the sclera of eyes, soles of feet, and palms of hands. How should the nurse document these findings? a. Normal b. Erythema c. Jaundice d. Ecchymosis

ANS: C Jaundice is defined as the yellow staining of the skin, usually by bile pigments. Yellow staining is not a normal appearance of the skin. Erythema is redness that results from increased blood flow to the area. Ecchymosis is large, diffuse areas, usually black and blue, caused by hemorrhage of blood into the skin.

Kimberly, age 3 years, is being admitted for about 1 week of hospitalization. Her parents tell the nurse that they are going to buy her "a lot of new toys, because she will be in the hospital." The nurse's reply should be based on an understanding of which concept? a. New toys make hospitalization easier. b. New toys are usually better than older ones for children of this age. c. At this age, children often need the comfort and reassurance of familiar toys from home. d. Buying new toys for a hospitalized child is a maladaptive way to cope with parental guilt.

ANS: C Parents should bring favorite items from home to be with the child. Young children associate inanimate objects with people who are significant in their lives. The favorite items will comfort and reassure the child. Because the parents left the objects, the preschooler knows the parents will return. New toys will not serve the purpose of familiar toys and objects from home. The parents may experience some guilt as a response to the hospitalization, but there is no evidence that it is maladaptive.

A school-age child, admitted for intravenous antibiotic therapy for osteomyelitis, reports difficulty in going to sleep at night. Which intervention should the nurse implement to assist the child in going to sleep at bedtime? a. Request a prescription for a sleeping pill. b. Allow the child to stay up late and sleep late in the morning. c. Create a schedule similar to the one the child follows at home. d. Plan passive activities in the morning and interactive activities right before bedtime.

ANS: C Many children obtain significantly less sleep in the hospital than at home; the primary causes are a delay in sleep onset and early termination of sleep because of hospital routines. One technique that can minimize the disruption in the child's routine is establishing a daily schedule. This approach is most suitable for non-critically ill school-age and adolescent children who have mastered the concept of time. It involves scheduling the child's day to include all those activities that are important to the child and nurse, such as treatment procedures, schoolwork, exercise, television, playroom, and hobbies. The school-age child with osteomyelitis would benefit from a schedule similar to the one followed at home. Requesting a prescription for a sleeping pill would be inappropriate and allowing the child to stay up late and sleep late would not be keeping the child in a routine followed at home. Passive activities in the morning and interactive activities at bedtime should be reversed; it would be better to keep the child active in the morning hours and plan quiet activities at bedtime.

A nurse is assessing a child with an unrepaired ventricular septal defect. Which heart sound does the nurse expect to assess? a. S3 b. S4 c. Murmur d. Physiologic splitting

ANS: C Murmurs are the sounds that are produced in the heart chambers or major arteries from the back-and-forth flow of blood. These are the sounds expected to be heard in a child with a ventricular septal defect because of the abnormal opening between the ventricles. S3 is a normal heart sound sometimes heard in children. S4 is rarely heard as a normal heart sound. If heard, medical evaluation is required. Physiologic splitting is the distinction of the two sounds in S2, which widens on inspiration. It is a significant normal finding.

Which action by the school nurse is important in the prevention of rheumatic fever? a. Encourage routine cholesterol screenings. b. Conduct routine blood pressure screenings. c. Refer children with sore throats for throat cultures. d. Recommend salicylates instead of acetaminophen for minor discomforts.

ANS: C Nurses have a role in prevention—primarily in screening school-age children for sore throats caused by group A β-hemolytic streptococci. They can achieve this by actively participating in throat culture screening or by referring children with possible streptococcal sore throats for testing. Cholesterol and blood pressure screenings do not facilitate the recognition and treatment of group A β-hemolytic streptococci. Salicylates should be avoided routinely because of the risk of Reye syndrome after viral illnesses.

What clinical manifestation is included in toxic shock syndrome? a. Severe hypertension b. Subnormal temperature c. Erythematous macular rash d. Papular rash over extremities

ANS: C One of the diagnostic criteria for toxic shock syndrome is a diffuse macular erythroderma. Hypotension is one of the manifestations. Fever of 38.9° C or higher is a characteristic. Desquamation of the palms and soles of the feet occurs in about 1 to 2 weeks.

Home care is being considered for a young child who is ventilator-dependent. Which factor is most important in deciding whether home care is appropriate? a. Level of parents' education b. Presence of two parents in the home c. Preparation and training of family d. Family's ability to assume all health care costs

ANS: C One of the essential elements is the family's training and preparation. The family must be able to demonstrate all aspects of care for the child. In many areas, it cannot be guaranteed that nursing care will be available on a continual basis, and the family will have to care for the child. The amount of formal education reached by the parents is not the important issue. The determinant is the family's ability to care adequately for the child in the home. At least two family members should learn and demonstrate all aspects of the child's care in the hospital, but it does not have to be two parents. Few families can assume all health care costs. Creative financial planning, including negotiating arrangements with the insurance company and/or public programs, may be required

An 8-year-old girl asks the nurse how the blood pressure apparatus works. What is the most appropriate nursing action? a. Ask her why she wants to know. b. Determine why she is so anxious. c. Explain in simple terms how it works. d. Tell her she will see how it works as it is used.

ANS: C School-age children require explanations and reasons for everything. They are interested in the functional aspect of all procedures, objects, and activities. It is appropriate for the nurse to explain how equipment works and what will happen to the child. A nurse should respond positively for requests for information about procedures and health information. By not responding, the nurse may be limiting communication with the child. The child is not exhibiting anxiety, just requesting clarification of what will be occurring. The nurse must explain how the blood pressure cuff works so that the child can then observe during the procedure.

During a routine health assessment, the nurse notes that an 8-month-old infant has significant head lag. Which is the nurse's most appropriate action? a. Teach parents appropriate exercises. b. Recheck head control at next visit. c. Refer child for further evaluation. d. Refer child for further evaluation if anterior fontanel is still open.

ANS: C Significant head lag after age 6 months strongly indicates cerebral injury and is referred for further evaluation. Reduction of head lag is part of normal development. Exercises will not be effective. The lack of achievement of this developmental milestone must be evaluated.

Parents of two school-age children with asthma ask the nurse, "What sports can our children participate in?" The nurse should recommend which sport? a. Soccer b. Running c. Swimming d. Basketball

ANS: C Swimming is well tolerated in children with asthma because they are breathing air fully saturated with moisture and because of the type of breathing required in swimming. Exercise-induced bronchospasm is more common in sports that involve endurance, such as soccer. Prophylaxis with medications may be necessary.

Which is the most appropriate vision acuity test for a child who is in preschool? a. Cover test b. Ishihara test c. HOTV chart d. Snellen letter chart

ANS: C The HOTV test consists of a wall chart of these letters. The child is asked to point to a corresponding card when the examiner selects one of the letters on the chart. The cover test determines ocular alignment. The Ishihara test is used for the detection of color blindness. The Snellen letter chart is usually used for older children.

When doing a nutritional assessment on a Hispanic family, the nurse learns that their diet consists mainly of vegetables, legumes, and starches. How should the nurse assess this diet? a. Indicates they live in poverty b. Is lacking in protein c. May provide sufficient amino acids d. Should be enriched with meat and milk

ANS: C The diet that contains vegetable, legumes, and starches may provide sufficient essential amino acids, even though the actual amount of meat or dairy protein is low. Many cultures use diets that contain this combination of foods. It is not indicative of poverty. Combinations of foods contain the essential amino acids necessary for growth. A dietary assessment should be done, but many vegetarian diets are sufficient for growth

It is now recommended that children with asthma who are taking long-term inhaled steroids should be assessed frequently because which disease or assessment findings may develop? a. Cough b. Osteoporosis c. Slowed growth d. Cushing syndrome

ANS: C The growth of children on long-term inhaled steroids should be assessed frequently to assess for systemic effects of these drugs. Cough is prevented by inhaled steroids. No evidence exists that inhaled steroids cause osteoporosis. Cushing syndrome is caused by long-term systemic steroids.

What is the single most important factor to consider when communicating with children? a. The child's physical condition b. Presence or absence of the child's parent c. The child's developmental level d. The child's nonverbal behaviors

ANS: C The nurse must be aware of the child's developmental stage to engage in effective communication. The use of both verbal and nonverbal communication should be appropriate to the developmental level. Although the child's physical condition is a consideration, developmental level is much more important. The parents' presence is important when communicating with young children but may be detrimental when speaking with adolescents. Nonverbal behaviors will vary in importance, based on the child's developmental level.

A nurse is preparing to complete an admission assessment on a 2-year-old child. The child is sitting on the parent's lap. Which technique should the nurse implement to complete the physical exam? a. Ask the parent to place the child in the hospital crib. b. Take the child and parent to the exam room. c. Perform the exam while the child is on the parent's lap. d. Ask the child to stand by the parent while completing the exam.

ANS: C The nurse should complete the exam while the child is on the parent's lap. For young children, particularly infants and toddlers, preserving parent-child contact is the best means of decreasing the need for or stress of restraint. The entire physical examination can be done in a parent's lap with the parent hugging the child for procedures such as an otoscopic examination. Placing the child in the crib, taking the child to the exam room, or asking the child to stand by the parent would separate the child from the parent and cause anxiety.

A child has a postoperative appendectomy incision covered by a dressing. The nurse has just completed a prescribed dressing change for this child. Which description is an accurate documentation of this procedure? a. Dressing change to appendectomy incision completed, child tolerated procedure well, parent present b. No complications noted during dressing change to appendectomy incision c. Appendectomy incision non-reddened, sutures intact, no drainage noted on old dressing, new dressing applied, procedure tolerated well by child d. No changes to appendectomy incisional area, dressing changed, child complained of pain during procedure, new dressing clean, dry and intact

ANS: C The nurse should document assessments and reassessments. Appearance of the incision described in objective terms should be included during a dressing change. The nurse should document patient's response and the outcomes of the care provided. In this example, these include drainage on the old dressing, the application of the new dressing, and the child's response. The other statements partially fulfill the requirements of documenting assessments and reassessments, patient's response, and outcome, but do not include all three.

Which action by the nurse demonstrates use of evidence-based practice (EBP)? a. Gathering equipment for a procedure b. Documenting changes in a patient's status c. Questioning the use of daily central line dressing changes d. Clarifying a physician's prescription for morphine

ANS: C The nurse who questions the daily central line dressing change is ascertaining whether clinical interventions result in positive outcomes for patients. This demonstrates EBP, which implies questioning why something is effective and whether a better approach exists. Gathering equipment for a procedure and documenting changes in a patient's status are practices that follow established guidelines. Clarifying a physician's prescription for morphine constitutes safe nursing care.

When communicating with other professionals, what is important for the nurse to do? a. Ask others what they want to know. b. Share everything known about the family. c. Restrict communication to clinically relevant information. d. Recognize that confidentiality is not possible.

ANS: C The nurse will need to share, through both oral and written communication, clinically relevant information with other involved health professionals. Asking others what they want to know and sharing everything known about the family is inappropriate. Patients have a right to confidentiality. The nurse is not permitted to share information about clients, except clinically relevant information that pertains to the child's care. Confidentiality permits the disclosure of information to other health professionals on a need-to-know basis.

A nurse is collecting subjective and objective information about target populations to diagnose problems based on community needs. This describes which step in the community nursing process? a. Planning b. Diagnosis c. Assessment d. Establishing objectives

ANS: C The nursing process stages are similar, whether the client is one child or a population of children. The assessment phase of the nursing process focuses on collecting subjective and objective data. Planning is the development of community-centered goals and objectives. Diagnosis is the identification of problems specific to the community.

A child is playing in the playroom. The nurse needs to obtain a child's blood pressure. Which is the appropriate procedure for obtaining the blood pressure? a. Take the blood pressure in the playroom. b. Ask the child to come to the exam room to obtain the blood pressure. c. Ask the child to return to his or her room for the blood pressure, then escort the child back to the playroom. d. Document that the blood pressure was not obtained because the child was in the playroom.

ANS: C The play room is a safe haven for children, free from medical or nursing procedures. The child can be returned to his or her room for the blood pressure and then escorted back to the playroom. The exam room is reserved for painful procedures that should not be performed in the child's hospital bed. Documenting that the blood pressure was not obtained because the child was in the playroom is inappropriate.

At about what age does the Babinski sign disappear? a. 4 months b. 6 months c. 1 year d. 2 years

ANS: C The presence of the Babinski reflex after about age 1 year, when walking begins, is abnormal. Four to 6 months is too young for the disappearance of the Babinski reflex. Persistence of the Babinski reflex requires further evaluation.

A nurse on a pediatric unit is practicing family-centered care. Which is most descriptive of the care the nurse is delivering? a. Taking over total care of the child to reduce stress on the family b. Encouraging family dependence on health care systems c. Recognizing that the family is the constant in a child's life d. Excluding families from the decision-making process

ANS: C The three key components of family-centered care are respect, collaboration, and support. Family-centered care recognizes the family as the constant in the child's life. Taking over total care does not include the family in the process and may increase stress instead of reducing stress. The family should be enabled and empowered to work with the health care system. The family is expected to be part of the decision-making process.

Which is the major cause of death for children older than 1 year? a. Cancer b. Heart disease c. Unintentional injuries d. Congenital anomalies

ANS: C Unintentional injuries (accidents) are the leading cause of death after age 1 year through adolescence. Congenital anomalies are the leading cause of death in those younger than 1 year. Cancer ranks either second or fourth, depending on the age group, and heart disease ranks fifth in the majority of the age groups.

The nurse is testing an infant's visual acuity. By what age should the infant be able to fix on and follow a target? a. 1 month b. 1 to 2 months c. 3 to 4 months d. 6 months

ANS: C Visual fixation and following a target should be present by ages 3 to 4 months. One to 2 months is too young for this developmental milestone. If the infant is not able to fix and follow by 6 months, further ophthalmologic evaluation is needed.

A 14-year-old boy is being admitted to the hospital for an appendectomy. Which roommate should the nurse assign with this patient? a. A 4-year-old boy post-appendectomy surgery b. A 6-year-old boy with pneumonia c. A 15-year-old boy admitted with a vasoocclusive sickle cell crisis d. A 12-year-old boy with cellulitis

ANS: C When a child is admitted, nurses follow several fairly universal admission procedures. The minimum considerations for room assignment are age, sex, and nature of the illness. Age grouping is especially important for adolescents. The 14-year-old boy being admitted to the unit after appendectomy surgery should be placed with a noninfectious child of the same sex and age. The 15-year-old child with sickle cell is the best choice. The 4-year-old post-appendectomy is too young, and the child with pneumonia is too young and possibly has an infectious process. The 12-year-old boy with cellulitis is the right age, but he has an infection (cellulitis).

The nurse is conducting discharge teaching about signs and symptoms of heart failure to parents of an infant with a repaired tetralogy of Fallot. Which signs and symptoms should the nurse include? (Select all that apply.) a. Warm flushed extremities b. Weight loss c. Decreased urinary output d. Sweating (inappropriate) e. Fatigue

ANS: C, D, E The signs and symptoms of heart failure include decreased urinary output, sweating, and fatigue. Other signs include pale, cool extremities, not warm and flushed, and weight gain, not weight loss.

Which clinical manifestation should the nurse expect to see as shock progresses in a child and becomes decompensated shock? (Select all that apply.) a. Thirst and diminished urinary output b. Irritability and apprehension c. Cool extremities and decreased skin turgor d. Confusion and somnolence e. Normal blood pressure and narrowing pulse pressure f. Tachypnea and poor capillary refill time

ANS: C, D, F Cool extremities, decreased skin turgor, confusion, somnolence, tachypnea, and poor capillary refill time are beginning signs of decompensated shock. Thirst, diminished urinary output, irritability, apprehension, normal blood pressure, and narrowing pulse pressure are signs of compensated shock.

Nursing interventions for the child after a cardiac catheterization should include which actions? (Select all that apply.) a. Allow ambulation as tolerated. b. Monitor vital signs every 2 hours. c. Assess the affected extremity for temperature and color. d. Check pulses above the catheterization site for equality and symmetry. e. Remove pressure dressing after 4 hours. f. Maintain a patent peripheral intravenous catheter until discharge.

ANS: C, F The extremity that was used for access for the cardiac catheterization must be checked for temperature and color. Coolness and blanching may indicate arterial occlusion. The child should have a patent peripheral intravenous line (PIV) to ensure adequate hydration. The child should remain on bed rest with the leg extended for a minimum of 4 hours. Initially vital signs are taken every 15 minutes, with emphasis on a heart rate counted for 1 minute. Pulses above the catheterization site should not be affected by the catheterization. Pulses distal to the site should be monitored. The pressure dressings should not be removed for 24 hours.

Pulses can be graded according to certain criteria. Which is a description of a normal pulse? a. 0 b. +1 c. +2 d. +3

ANS: D A normal pulse is described as +3. A pulse that is easy to palpate and not easily obliterated with pressure is considered normal. A pulse graded 0 is not palpable. A pulse graded +1 is difficult to palpate, thready, weak, and easily obliterated with pressure. A pulse graded +2 is difficult to palpate and may be easily obliterated with pressure.

Cystic fibrosis (CF) is suspected in a toddler. Which test is essential in establishing this diagnosis? a. Bronchoscopy b. Serum calcium c. Urine creatinine d. Sweat chloride test

ANS: D A sweat chloride test result greater than 60 mEq/L is diagnostic of CF. Bronchoscopy, although helpful for identifying bacterial infection in children with CF, is not diagnostic. Serum calcium is normal in children with CF. Urine creatinine is not diagnostic of CF.

A nurse is planning a teaching session for parents of preschool children. Which statement explains why the nurse should include information about morbidity and mortality? a. Life span statistics are included in the data. b. It explains effectiveness of treatment. c. Cost-effective treatment is detailed for the general population. d. High-risk age groups for certain disorders or hazards are identified.

ANS: D Analysis of morbidity and mortality data provides the parents with information about which groups of individuals are at risk for which health problems. Life span statistics is a part of the mortality data. Treatment modalities and cost are not included in morbidity and mortality data.

Which type of shock is characterized by a hypersensitivity reaction causing massive vasodilation and capillary leaks, which may occur with drug or latex allergy? a. Neurogenic b. Cardiogenic c. Hypovolemic d. Anaphylactic

ANS: D Anaphylactic shock results from extreme allergy or hypersensitivity to a foreign substance. Neurogenic shock results from loss of neuronal control, such as the interruption of neuronal transmission that occurs from a spinal cord injury. Cardiogenic shock is decreased cardiac output. Hypovolemic shock is a reduction in the size of the vascular compartment, decreasing blood pressure, and low central venous pressure.

How does the nurse assess a child's capillary refill time? a. Inspecting the chest b. Auscultating the heart c. Palpating the apical pulse d. Palpating the skin to produce a slight blanching

ANS: D Capillary refill time is assessed by pressing lightly on the skin to produce blanching, and then noting the amount of time it takes for the blanched area to refill. Inspecting the chest, auscultating the heart, and palpating the apical pulse will not provide an assessment of capillary refill time.

Which is a common, serious complication of rheumatic fever? a. Seizures b. Cardiac arrhythmias c. Pulmonary hypertension d. Cardiac valve damage

ANS: D Cardiac valve damage is the most significant complication of rheumatic fever. Seizures, cardiac arrhythmias, and pulmonary hypertension are not common complications of rheumatic fever.

A clinic nurse is planning a teaching session about childhood obesity prevention for parents of school-age children. The nurse should include which associated risk of obesity in the teaching plan? a. Type I diabetes b. Respiratory disease c. Celiac disease d. Type II diabetes

ANS: D Childhood obesity has been associated with the rise of type II diabetes in children. Type I diabetes is not associated with obesity and has a genetic component. Respiratory disease is not associated with obesity, and celiac disease is the inability to metabolize gluten in foods and is not associated with obesity.

Which frequency is recommended for childhood skin testing for tuberculosis (TB) using the Mantoux test? a. Every year for all children older than 2 years b. Every year for all children older than 10 years c. Every 2 years for all children starting at age 1 year d. Periodically for children who reside in high-prevalence regions

ANS: D Children who reside in high-prevalence regions for TB should be tested every 2 to 3 years. Annual testing is not necessary. Testing is not necessary unless exposure is likely or an underlying medical risk factor is present.

The parents of a young child with heart failure tell the nurse that they are "nervous" about giving digoxin (Lanoxin). The nurse's response should be based on which statement? a. It is a safe, frequently used drug. b. It is difficult to either overmedicate or undermedicate with digoxin. c. Parents lack the expertise necessary to administer digoxin. d. Parents must learn specific, important guidelines for administration of digoxin.

ANS: D Digoxin has a narrow therapeutic range. The margin of safety between therapeutic, toxic, and lethal doses is very small. Specific guidelines are available for parents to learn how to administer the drug safely and to monitor for side effects. Digoxin is a frequently used drug, but it has a narrow therapeutic range. Small amounts of the liquid are given to infants, making it easy to overmedicate or undermedicate. Parents may lack the necessary expertise to administer the drug at first, but with discharge preparation, they should be prepared to administer the drug safely.

The nurse is having difficulty communicating with a hospitalized 6-year-old child. What technique might be most helpful? a. Suggest that the child keep a diary. b. Suggest that the parent read fairy tales to the child. c. Ask the parent if the child is always uncommunicative. d. Ask the child to draw a picture.

ANS: D Drawing is one of the most valuable forms of communication. Children's drawings tell a great deal about them because they are projections of the child's inner self. It would be difficult for a 6-year-old child who is most likely learning to read to keep a diary. Parents reading fairy tales to the child is a passive activity involving the parent and child. It would not facilitate communication with the nurse. The child is in a stressful situation and is probably uncomfortable with strangers.

Pancreatic enzymes are administered to the child with cystic fibrosis (CF). What nursing considerations should be included? a. Do not administer pancreatic enzymes if the child is receiving antibiotics. b. Decrease dose of pancreatic enzymes if the child is having frequent, bulky stools. c. Administer pancreatic enzymes between meals if at all possible. d. Pancreatic enzymes can be swallowed whole or sprinkled on a small amount of food taken at the beginning of a meal.

ANS: D Enzymes may be administered in a small amount of cereal or fruit at the beginning of a meal or swallowed whole. Pancreatic enzymes are not a contraindication for antibiotics. The dosage of enzymes should be increased if the child is having frequent, bulky stools. Enzymes should be given just before meals and snacks.

Which term best describes the emotional attitude that one's own ethnic group is superior to others? a. Culture b. Ethnicity c. Superiority d. Ethnocentrism

ANS: D Ethnocentrism is the belief that one's way of living and behaving is the best way. This includes the emotional attitude that the values, beliefs, and perceptions of one's ethnic group are superior to those of others. Culture is a pattern of assumptions, beliefs, and practices that unconsciously frames or guides the outlook and decisions of a group of people. A culture is composed of individuals who share a set of values, beliefs, and practices that serves as a frame of reference for individual perception and judgments. Ethnicity is an affiliation of a set of persons who share a unique cultural, social, and linguistic heritage. Superiority is the state or quality of being superior; it does not include ethnicity.

The nurse is caring for a school-age child who has had a cardiac catheterization. The child tells the nurse that the bandage is "too wet." The nurse finds the bandage and bed soaked with blood. What is the priority nursing action? a. Notify physician b. Apply new bandage with more pressure c. Place the child in Trendelenburg position d. Apply direct pressure above catheterization site

ANS: D If bleeding occurs, direct continuous pressure is applied 2.5 cm (1 inch) above the percutaneous skin site to localize pressure over the vessel puncture. Notifying a physician and applying a new bandage can be done after pressure is applied. The nurse can have someone else notify the physician while the pressure is being maintained. It is not a helpful intervention to place the girl in the Trendelenburg position. It would increase the drainage from the lower extremities.

Seventy-two hours after cardiac surgery, a young child has a temperature of 101° F. Which action should the nurse take? a. Keep child warm with blankets. b. Apply a hypothermia blanket. c. Record temperature on nurses' notes. d. Report findings to physician.

ANS: D In the first 24 to 48 hours after surgery, the body temperature may increase to 37.7° C (100° F) as part of the inflammatory response to tissue trauma. If the temperature is higher or continues after this period, it is most likely a sign of an infection and immediate investigation is indicated. Blankets should be removed from the child to keep the temperature from increasing. Hypothermia blanket is not indicated for this level of temperature. The temperature should be recorded, but the physician must be notified for evaluation. Suctioning should be done only as indicated, not on a routine basis. The child should be suctioned for no more than 5 seconds at one time. Symptoms of respiratory distress are avoided by using appropriate technique.

A mother brings 6-month-old Eric to the clinic for a well-baby checkup. She comments, "I want to go back to work, but I don't want Eric to suffer because I'll have less time with him." The nurse's most appropriate answer would be which statement? a. "I'm sure he'll be fine if you get a good babysitter." b. "You will need to stay home until Eric starts school." c. "You should go back to work so Eric will get used to being with others." d. "Let's talk about the child care options that will be best for Eric."

ANS: D Let's talk about the child care options that will be best for Eric is an open-ended statement that will assist the mother in exploring her concerns about what is best for both her and Eric. I'm sure he'll be fine if you get a good babysitter, You will need to stay home until Eric starts school, and You should go back to work so Eric will get used to being with others are directive statements. They do not address the effect of her working on Eric.

How is chronic otitis media with effusion (OME) differentiated from acute otitis media (AOM)? a. A fever as high as 40° C (104° F) b. Severe pain in the ear c. Nausea and vomiting d. A feeling of fullness in the ear

ANS: D OME is characterized by feeling of fullness in the ear or other nonspecific complaints. Fever is a sign of AOM. OME does not cause severe pain. This may be a sign of AOM. Nausea and vomiting are associated with otitis media.

Parents have understood teaching about prevention of childhood otitis media if they make which statement? a. "We will only prop the bottle during the daytime feedings." b. "Breastfeeding will be discontinued after 4 months of age." c. "We will place the child flat right after feedings." d. "We will be sure to keep immunizations up to date."

ANS: D Parents have understood the teaching about preventing childhood otitis media if they respond they will keep childhood immunizations up to date. The child should be maintained upright during feedings and after. Otitis media can be prevented by exclusively breastfeeding until at least 6 months of age. Propping bottles is discouraged to avoid pooling of milk while the child is in the supine position.

Which is an appropriate nursing intervention when caring for an infant with an upper respiratory tract infection and elevated temperature? a. Give tepid water baths to reduce fever. b. Encourage food intake to maintain caloric needs. c. Have child wear heavy clothing to prevent chilling. d. Give small amounts of favorite fluids frequently to prevent dehydration.

ANS: D Preventing dehydration by small frequent feedings is an important intervention in the febrile child. Tepid water baths may induce shivering, which raises temperature. Food should not be forced; it may result in the child vomiting. The febrile child should be dressed in light, loose clothing.

What is a common initial reaction of parents to illness or injury and hospitalization in their child? a. Anger b. Fear c. Depression d. Helplessness

ANS: D Recent research has identified common themes among parents whose children were hospitalized, including feeling an overall sense of helplessness, questioning the skills of staff, accepting the reality of hospitalization, needing to have information explained in simple language, dealing with fear, coping with uncertainty, and seeking reassurance from caregivers. Anger or guilt is usually the second reaction stage. Fear, anxiety, and frustrations also are common feelings. Parents may finally react with some form of depression related to the physical and emotional exhaustion associated with a hospitalized child.

Which is the most common causative agent of bacterial endocarditis? a. Staphylococcus albus b. Streptococcus hemolyticus c. Staphylococcus albicans d. Streptococcus viridans

ANS: D S. viridans is the most common causative agent in bacterial (infective) endocarditis. Staphylococcus albus, Streptococcus hemolyticus, and Staphylococcus albicans are not common causative agents.

Which drug is usually given first in the emergency treatment of an acute, severe asthma episode in a young child? a. Ephedrine b. Theophylline c. Aminophylline d. Short-acting β2 agonists

ANS: D Short-acting β2 agonists are the first treatment in an acute asthma exacerbation. Ephedrine is not helpful in acute asthma exacerbations. Theophylline is unnecessary for treating asthma exacerbations. Aminophylline is not helpful for acute asthma exacerbation.

When palpating the child's cervical lymph nodes, the nurse notes that they are tender, enlarged, and warm. What is the best explanation for this? a. Some form of cancer b. Local scalp infection common in children c. Infection or inflammation distal to the site d. Infection or inflammation close to the site

ANS: D Small nontender nodes are normal. Tender, enlarged, and warm lymph nodes may indicate infection or inflammation close to their location. Tender lymph nodes are not usually indicative of cancer. A scalp infection would usually not cause inflamed lymph nodes. The lymph nodes close to the site of inflammation or infection would be inflamed.

A nurse is preparing to test a school-age child's vision. Which eye chart should the nurse use? a. Denver Eye Screening Test b. Allen picture card test c. Ishihara vision test d. Snellen letter chart

ANS: D The Snellen letter chart, which consists of lines of letters of decreasing size, is the most frequently used test for visual acuity for school-age children. Single cards (Denver—letter E; Allen—pictures) are used for children ages 2 years and older who are unable to use the Snellen letter chart. The Ishihara vision test is used for color vision.

At what age should the nurse expect the anterior fontanel to close? a. 2 months b. 2 to 4 months c. 6 to 8 months d. 12 to 18 months

ANS: D The anterior fontanel normally closes between ages 12 and 18 months. Two to 8 months is too early. The expected closure of the anterior fontanel occurs between ages 12 and 18 months; if it closes between ages 2 and 8 months, the child should be referred for further evaluation.

Which explanation regarding cardiac catheterization is appropriate for a preschool child? a. Postural drainage will be performed every 4 to 6 hours after the test. b. It is necessary to be completely "asleep" during the test. c. The test is short, usually taking less than 1 hour. d. When the procedure is done, you will have to keep your leg straight for at least 4 hours.

ANS: D The child's leg will have to be maintained in a straight position for approximately 4 hours. Younger children can be held in the parent's lap with the leg maintained in the correct position. Postural drainage will not be performed unless the child has corresponding pulmonary problems. The child should be sedated to lie still, but being completely asleep is not necessary. The test will vary in length of time from start to finish.

The mother of a toddler yells to the nurse, "Help! He is choking to death on his food." The nurse determines that lifesaving measures are necessary based on which symptom? a. Gagging b. Coughing c. Pulse over 100 beats/min d. Inability to speak

ANS: D The inability to speak is indicative of a foreign-body airway obstruction of the larynx. Abdominal thrusts are needed for treatment of the choking child. Gagging indicates irritation at the back of the throat, not obstruction. Coughing does not indicate a complete airway obstruction. Tachycardia may be present for many reasons.

Which is now referred to as the "new morbidity"? a. Limitations in the major activities of daily living b. Unintentional injuries that cause chronic health problems c. Discoveries of new therapies to treat health problems d. Behavioral, social, and educational problems that alter health

ANS: D The new morbidity reflects the behavioral, social, and educational problems that interfere with the child's social and academic development. It is currently estimated that the incidence of these issues is from 5% to 30%. Limitations in major activities of daily living and unintentional injuries that result in chronic health problems are included in morbidity data. Discovery of new therapies would be reflected in changes in morbidity data over time.

A nurse is establishing several health programs, such as bicycle safety, to improve the health status of a target population. This describes which step in the community nursing process? a. Planning b. Evaluation c. Assessment d. Implementation

ANS: D The nurse working with the community to put into practice a program to reach community goals is the implementation phase of the community nursing process. Planning involves designing the program to meet community-centered goals. The evaluation stage would determine the effectiveness of the program. During the assessment phase, the nurse would identify the resources necessary and the barriers that would interfere with implementation.

The nurse is caring for a 10-month-old infant with respiratory syncytial virus (RSV) bronchiolitis. Which intervention should be included in the child's care? (Select all that apply.) a. Place in a mist tent. b. Administer antibiotics. c. Administer cough syrup. d. Encourage the child to drink 8 ounces of formula every 4 hours. e. Cluster care to encourage adequate rest. f. Place on noninvasive oxygen monitoring.

ANS: D, E, F Hydration is important in children with RSV bronchiolitis to loosen secretions and prevent shock. Clustering of care promotes periods of rest. The use of noninvasive oxygen monitoring is recommended. Mist tents are no longer used. Antibiotics do not treat illnesses with viral causes. Cough syrup suppresses clearing of respiratory secretions and is not indicated for young children.

The National children's Study is the largest prospective, long-term study of children's health and development in the United States. Which of these options are the goals of this study? Select all that apply. a. Ensure that every child is immunized at the appropriate age. b. Provide information for families to eradicate unhealthy diets, dental caries, and childhood obesity. c. Enlist and help of school lunch programs to reach the goal of vegetables and fruits as 30% of each lunch d. Significantly reduce violence, substance abuse, and mental health disorders among the nation's children. e. Decrease tardiness and truancy and increase the high school graduation rate in each state over the next 5 years.

B & D: Help improve youth health by prevention and early detection

Duvall's Developmental Stages of the Family include which of the following? Select all that apply. a. Stages an individual progresses through in their moral and spiritual development b. Stages families progress through in adulthood c. Stages that designate how parenting progresses as a child develops d. Stages that designate appropriate discipline related to developmental stages e. Stages that describe the journey a couple will take as their children mature

B, C, & E

You tell the parent of a 4-year-old patient being admitted that you need to ask some questions. She asks, "Why do you have to ask so many questions?" Which explanations should you offer? Select all that apply. a. "It is something we are required to do for every child who is hospitalized." b. "By learning about your child's routines, we can try to minimize some of the changes he will be going through." c. "Knowing more about your child can help predict how the hospital stay will go and will also help us choose a good roommate for him when more children arrive at the hospital." d. "Gaining more information about your child, such as current medications she is taking, will help us provide the best care." e. "This will give you an opportunity to ask questions as well."

a. "It is something we are required to do for every child who is hospitalized." b. "By learning about your child's routines, we can try to minimize some of the changes he will be going through." e. "This will give you an opportunity to ask questions as well."

When discharging the pediatric patient from the outpatient setting, the nurse knows which of the following responses indicate a need for more teaching? Select all that apply. a. "The physician said my son can have clear liquids when we return home, which would include Jell-O, pudding, and apple juice." b. "The other nurse explained that I can use other things to help with the pain, such as distraction (reading a book, music, or a movie), after the pain medication is given." c. "I can get my child's prescription tomorrow, so I can go to my regular pharmacy where they can explain the medication to me." d. "I am waiting for my husband to come so he can drive us, and I can watch my son in the car on the way home." e. "I understand that I will be contacted tomorrow for follow-up on my child but that I should not hesitate to call if I have any concerns before then."

a. "The physician said my son can have clear liquids when we return home, which would include Jell-O, pudding, and apple juice." c. "I can get my child's prescription tomorrow, so I can go to my regular pharmacy where they can explain the medication to me."

A 3-month-old infant is seen in the clinic with the following symptoms: irritability, crying, refusal to nurse for more than 2 to 3 minutes, rhinitis, and a rectal temperature of 101.8° F (38.8° C). The labor, delivery, and postpartum history for this term infant is unremarkable. The nurse anticipates a diagnosis of: a. Acute otitis media (AOM) b. Otitis media with effusion (OME) c. Otitis externa d. Respiratory syncytial virus (RSV)

a. Acute otitis media (AOM)

Which heart defect and hemodynamic change pairing is correct? a. Aortic stenosis and obstruction to blood flow out of the heart b. Ventricular septal defect (VSD) and decreased pulmonary blood flow c. Tricuspid atresia and increased pulmonary blood flow d. Atrioventricular (AV) canal and mixed blood flow, in which saturated and desaturated blood mix within the heart or great arteries

a. Aortic stenosis and obstruction to blood flow out of the heart

A 2-month-old formerly healthy infant born at term is seen in the urgent care clinic with intercostal retractions, respiratory rate of 62, heart rate of 128, refusal to breastfeed, abundant nasal secretions, and a pulse oximeter reading of 88% in room air. The diagnosis of respiratory syncytial virus (RSV) is made, and a bronchodilator is administered. The infant's oxygen saturation (SaO2) remains 95% in room air, and the respiratory rate is 54, with intercostal retractions; heart rate is 120 bpm. After 2 hours of observation and an intravenous (IV) bolus of fluids, the infant is being discharged home. The nurse provides which of the following home care instructions for this infant? Select all that apply. a. Continue breastfeeding infant. b. Discontinue breastfeeding and administer Pedialyte for 24 hours. c. Observe infant for labored breathing or apnea (cessation of breathing). d. Instill normal saline drops in both nares and suction thoroughly before feeding and before placing to sleep. e. Place infant to sleep on his side with the head of bed slightly elevated to facilitate breathing. f. Keep the infant out of daycare or nursery.

a. Continue breastfeeding infant. c. Observe infant for labored breathing or apnea (cessation of breathing). d. Instill normal saline drops in both nares and suction thoroughly before feeding and before placing to sleep. f. Keep the infant out of daycare or nursery.

When assessing BP in a child: a. Knowledge of normal mean is important: newborn, 65/41; 1 month to 2 years old, 95/58; and 2 to 5 years old, 101/57. b. Cuff size is the most important variable and should be measured using limb length. c. The child is considered normotensive if the BP is below the 95th percentile. d. Check upper- and lower-extremity BP to look for abnormalities, such as aortic stenosis, which causes lower-extremity BP to be higher than upper-extremity BP.

a. Knowledge of normal mean is important: newborn, 65/41; 1 month to 2 years old, 95/58; and 2 to 5 years old, 101/57.

Play is children's work, even in the hospital. Which of the following are functions of play? Select all that apply. a. Provides diversion and brings about relaxation b. Keeps the child occupied and directs concerns away from himself or herself c. Helps the child feel more secure in a strange environment d. Lessens the stress of separation and the feeling of homesickness e. Provides a means for release of tension and expression of feelings f. Allows the parents to have a break from the unit for a respite period

a. Provides diversion and brings about relaxation c. Helps the child feel more secure in a strange environment d. Lessens the stress of separation and the feeling of homesickness e. Provides a means for release of tension and expression of feelings

Growth measurement is a key element in children of their health status. One measurement for height is linear growth measurement. What should the nurse do to perfect this technique? Select all that apply. a. Understand the difference in measurement for children who can stand alone and for those who must lie recumbent. b. Use a length board and footboard or a stadiometer, which is the best technique, or use a tape measure. c. Two measurers are usually required for a recumbent child, although one measurer may be sufficient for a cooperative child. d. Reposition the child and repeat the procedure. Measure at least twice (ideally three times). Average the measurements for the final value. e. Demonstrate competency when measuring the growth of infants, children, and adolescents. Refresher sessions should be taken when a lack of standardization occurs.

a. Understand the difference in measurement for children who can stand alone and for those who must lie recumbent. d. Reposition the child and repeat the procedure. Measure at least twice (ideally three times). Average the measurements for the final value. e. Demonstrate competency when measuring the growth of infants, children, and adolescents. Refresher sessions should be taken when a lack of standardization occurs.

These general approaches can be helpful when performing a physical exam. Select all that apply. a. With toddlers, restraint may be necessary, and requesting a parent's assistance is appropriate. b. When examining a preschooler, giving a choice of which parts to examine may be helpful in gaining the child's cooperation. c. With a school-age child, it is always best to have the parents present when examining. d. Giving explanations about body systems can make adolescents nervous due to their egocentricities. e. An infant physical exam is done head to toe, similarly to the adult.

a. With toddlers, restraint may be necessary, and requesting a parent's assistance is appropriate. b. When examining a preschooler, giving a choice of which parts to examine may be helpful in gaining the child's cooperation.

While interviewing parents who have just arrived in the health care clinic, the nurse begins the interview. Which statement below involves therapeutic communication techniques? Select all that apply. a. Allow the parents to direct the conversation so that they feel comfortable and in control. b. Use broad, open-ended questions so that parents can feel open to discuss issues. c. Redirect by asking guided questions to keep the parents on task. d. Use careful listening, which relies on the use of clues and verbal leads to help move the conversation along. e. Ask carefully worded, detailed questions to get accurate information.

b. Use broad, open-ended questions so that parents can feel open to discuss issues. c. Redirect by asking guided questions to keep the parents on task. d. Use careful listening, which relies on the use of clues and verbal leads to help move the conversation along.


Conjuntos de estudio relacionados

MUSIC 351: Music Of Motown Part 2

View Set

Qualified Plans/Federal Tax Considerations For Life Insurance and Annuities

View Set

Chapter 3: Charismatic and Transformational Leadership

View Set

Accounting 307 Chapter Database Design

View Set

Earth Science Test 3 : Igneous Rocks or Something Like That

View Set

PhysioPsych Chapter 2: Nerve Cells and Nerve Impulses

View Set